SlideShare a Scribd company logo
1 of 40
Download to read offline
ORIGINAL COPY
1 of 40
NIZAM’S INSTITUTE OF MEDICAL SCIENCES
Punjagutta, Hyderabad – 500 082, A.P., India.
Set: C
NIMSET 2013
Question Booklet
Entrance Examination for Admission to
Postgraduate courses in MD/MS
24 February 2013 Marks 150 Time: 2 hours 30 min
• Please verify whether the number printed on this question booklet corresponds to
your hall-ticket number
• Please read the instructions on the OMR-Answer sheet carefully
• Each question in this booklet is followed by four alternative answers
Select the one that is most appropriate and darken the corresponding
Oval with HB pencil in the OMR-Answer sheet
• Please return this question booklet along with your OMR-Answer sheet
• Answer sheets not accompanied with the question booklets will not be corrected
• Wrong answers carry negative marks
Q1. The following drug does not act as treatment for acinetobacter infections
ORIGINAL COPY
2 of 40
A) Sulbactam
B) Ertapenem
C) Doripenem
D) Meropenem
Q2. A 24 year old girl is brought to emergency, with history of consuming unknown
substance. She complains of headache, giddiness and tinnitus. Her lips ,mouth and tongue are
corroded, white and hardened. Urine output is scanty and black in color. The most probable
substance consumed is
A) Sulfuric acid
B) Carbolic acid
C) Hydrochloric acid
D) Oxalic acid
Q3. A child with an absent thumb and hypoplastic radius has also required treatment for a
symptomatic arrhythmia and VSD. What is the most likely underlying unifying diagnosis?
A) Down syndrome
B) Chondroectodermal dysplasia (Ellis-van-Crevald syndrome)
C) Holt-Oram syndrome
D) Hailey-Hailey syndrome
Q4. Labyrinthine artery mostly arises from
A) Posterior inferior cerebellar artery (PICA)
B) Anterior inferior cerebellar artery (AICA)
C) Basilar artery
D) Vertebral Artery
Q5. Hebra nose is seen in
A) Leprosy
B) Rhinosporidiosis
C) Rhinitis medicamentosa
D) Rhinoscleroma
Q6. Complication/complications of ulcerative blepharitis:
1. Madarosis
ORIGINAL COPY
3 of 40
2. Fibrosis
3. Trichiasis
4. Optic neuritis
A) Only 1,2 and 3 are correct
B) Only 1 and 3 are correct
C) Only 2 and 4 are correct
D) Only 4 is correct
Q7. A 26 year old lady came with the complaints of fever with altered sensorium and reduced
urine output of 1 week duration. On evaluation her hemoglobin was 7 gm/dl, platelet count of
30,000/cu.mm with peripheral smear showing schistocytes. Serum creatinine was 3mg/dl.
Prothrombin time and activated partial thromboplastin time were normal. Enzyme most likely to
be deficient in this situation is
A) Hexokinase
B) Pyruvate kinase
C) ADAMTS 13
D) Glucose-6 phosphate dehydrogenase
Q8. A ten year old girl presented to the emergency medicine department with vague upper
abdominal pain and jaundice. On examination she was febrile and a tender mass was felt in the
right upper quadrant of the abdomen without signs of peritonitis. Laboratory studies revealed
raised serum bilirubin and serum alkaline phosphatase. The most probable diagnosis is
A) Perforated duodenal ulcer
B) Meckels diverticulum with sepsis
C) Cholelithiasis and choledocholithiasis
D) Choledochal cyst
Q9. Transmission of Clostridium difficile in clinical practice can be prevented by
1. Use of alcohol hand gels
2. Use of gloves by health care workers
3. Increasing the use of 2nd
and 3rd
generation cephalosporins
4. Use of hypochlorite solution for environmental decontamination
A) Only 1, 2 and 3 are correct
B) Only 1 and 3 are correct
C) Only 2 and 4 are correct
D) Only 4 is correct
Q10. Regarding 'Wernicke's encephalopathy'
Assertion: In acute presentation, patient has nystagmus, confusion,
ophthalmoplegia and ataxia
Reason: Caused by deficiency of thiamine
ORIGINAL COPY
4 of 40
A) Both Assertion and Reason are true and Reason is the correct explanation for
Assertion
B) Both Assertion and Reason are true and Reason is not the correct explanation for
Assertion
C) Assertion is true, but Reason is false
D) Assertion is false, but Reason is true
Q11. Regarding amiodarone
Assertion: it is indicated for therapy in patients with recurrent ventricular
tachycardia or fibrillation.
Reason: decreases refractoriness of all cardiac tissues.
A) Both Assertion and Reason are true, and Reason is the correct explanation for
Assertion
B) Both Assertion and Reason are true, and Reason is not the correct explanation
for Assertion
C) Assertion is true, but Reason is false
D) Assertion is false, but Reason is true
Q12. Antiretroviral therapy should be initiated
1) In all those who are HIV infected with CD4 count ≤ 350 cells/cu.mm
2) In HIV infected individuals with concurrent Hepatitis B virus infection
irrespective of CD4 count
3) With at least three drugs simultaneously for entire duration of treatment.
4) In all HIV infected adults and children irrespective of CD4 count.
A) 1, 2 & 3 are correct
B) Only 1 & 3 are correct
C) Only 2 & 4 are correct
D) Only 4 is correct
Q13. Regarding the management of myxedema coma
Assertion: parenteral hydrocortisone should be administered
Reason: there is concomittant autoimmune failure of adrenal glands in all patients with
myxedema coma
ORIGINAL COPY
5 of 40
A) Both Assertion and Reason are true and Reason is the correct explanation for the
Assertion
B) Both Assertion and Reason are true and Reason is not the correct explanation for
the Assertion
C) Assertion is true but Reason is false
D) Assertion is false but Reason is true
Q14. Cisplatin, an anticancer chemotherapeutic agent, is a
A) Alkylating agent
B) Antimetabolite
C) Antitumor antibiotic
D) Tyrosine kinase inhibitor
Q15. Which of these is not an inhibitor of fibrinolysis
A) Plasminogen Activator Inhibitor
B) Tissue Plasminogen Activator
C) α 2 - Antiplasmin
D) Thrombin Activatable Fibrinolysis Inhibitor
Q16. A patient with known ulcerative colitis was admitted as an emergency. The plain
abdominal radiograph showed dilatation and mucosal islands in keeping with toxic megacolon.
In which part of the large bowel are these changes most commonly seen?
A) Caecum
B) Ascending colon
C) Transverse colon
D) Descending colon
Q17. Regarding management of prosthetic joint infections
A) Prosthesis need not be replaced in most cases to clear the infection
B) Treatment includes surgery and high doses of parenteral antibiotics for up to 6
weeks
C) Implantation of the new prosthesis can be done immediately
D) Addition of rifampicin is not useful in patients with staphylococcal prosthetic
joint infection
Q18. Regarding treatment of yeast and dermatophyte fungal infection of skin
Assertion: Ketoconazole is an effective drug but reserved for severe infections
Reason: It can cause hepatotoxicity and thrombocytopenia.
ORIGINAL COPY
6 of 40
A) Both Assertion and Reason are true and Reason is correct explanation of
Assertion.
B) Both Assertion and Reason are true and Reason is not correct explanation of
Assertion.
C) Assertion is true and Reason is false
D) Assertion is false and Reason is true
Q19. Which of the following is not true regarding a case-control study
A) It is useful in studying rare diseases
B) It can measure incidence rates
C) It is useful in studying multiple risk factors
D) It is a retrospective study
Q20. In the graph shown below depicting the effect of increasing minimum alveolar
concentration (MAC)of an inhalational anesthetic agent on different components of anesthesia,
the components 1,2, and 3 correspond to which one of the following respectively
A) Unconsciousness, immobility, amnesia
B) Unconsciousness, amnesia, immobility,
C) Immobility, unconsciousness, amnesia,
D) Amnesia, unconsciousness, immobility
Q21. Which of the following correctly describes neurocysticercosis
A) Infection by Taenia solium and epilepsy
B) Infection by Echinoccocus and cyst formation
C) Always associated with immunodeficient state
ORIGINAL COPY
7 of 40
D) Formation of pyogenic brain abscess
Q22. A part of an mRNA is shown below. A mutation of the template DNA results in a change
in codon 66 to GGA. The type of mutation is
63 64 65 66 67
GAC CAG UAG GGC UAA
A) Missense
B) Silent
C) Nonsense
D) Frame shift
Q23. True statement/statements regarding MRSA
1. Mechanism of resistance is by production of beta lactamase.
2. It is susceptible to 3rd
and greater generation cephalosporins.
3. Beta-lactam and beta lactamase inhibitor combinations are effective against MRSA.
4. It is resistant to cloxacillin.
A) Only 1, 2 and 3 are correct
B) Only 1 and 3 are correct
C) Only 2 and 4 are correct
D) Only 4 is correct
Q24. Drugs that carry risk of clinical hemolysis in persons with G6PD deficiency include
1. Carbamazepine
2. Acetaminophen
3. Phenytoin
4. Nalidixic acid
A) Only 1, 2 and 3 are correct
B) Only 1 and 3 are correct
C) Only 2 and 4 are correct
D) Only 4 is correct
Q25. Regarding seminoma of the testes, the following statement is correct
A) Peak incidence of seminoma is in the second decade of life
B) Active lymphocytic infiltration of tumour suggests good host response and better
outcome.
ORIGINAL COPY
8 of 40
C) Early suspicion of the disease can be made out by inguinal lymphadenitis.
D) Surgery is the treatment of choice.
Q26. Low molecular weight (LMW) heparins have replaced continuous heparin (UFH) infusions
in management of thromboembolic conditions as
1) aPTT monitoring is not required for LMW heparins.
2) LMW Heparins can be given in patients with severe renal failure.
3) LMW Heparins have long biological half-lives
4) Heparin induced thrombocytopenia does not occur with LMW Heparins
A) 1, 2 & 3 are correct
B) Only 1 & 3 are correct
C) Only 2 & 4 are correct
D) Only 4 is correct
Q27. The complication of Uterine myoma that develops most frequently during pregnancy is,
A) Calcareous degeneration
B) Hyaline degeneration
C) Red degeneration
D) Cystic degeneration
Q28. A 68-year old male presented with easy fatigability, weight loss and anorexia. He was
found to have generalized lymphadenopathy and mild hepatosplenomegaly. His Hemoglobin was
12.0gm/dl, TLC- 130,000/cmm and platelets were normal. The blood smear is depicted in the
picture below
ORIGINAL COPY
9 of 40
What is the likely diagnosis?
A) Hairy cell leukemia
B) Chronic Myeloid leukemia
C) Chronic lymphocytic leukemia
D) Idiopathic myelofibrosis
Q29. The condition in which muscles are contracted during life, become stiff and rigid
immediately after death, without passing into the stage of primary relaxation is
A) Cadaveric Rigidity
B) Heat Stiffening
C) Cold Stiffening
D) Cadaveric Spasm
Q30. Regarding management of diabetic ketoacidosis
Assertion: after the initial bolus of normal saline and hemodynamic stability are
achieved intravenous fluids should be switched to 0.45% saline
Reason: 0.45% saline reduces hypernatremia
A) Both Assertion and Reason are true, and Reason is the correct explanation for
Assertion
B) Both Assertion and Reason are true, and Reason is not the correct explanation
for Assertion
C) Assertion is true, but Reason is false
D) Assertion is false, but Reason is true
Q31. The pain experienced by the woman who shed endometrium as a cast at the time of
menstruation is called
A) Spasmodic dysmenorrhea
ORIGINAL COPY
10 of 40
B) Congestive dysmenorrhea
C) Membranous dysmenorrhea
D) Unilateral dysmenorrhea
Q32. Regarding Ricocheting of a bullet, which of following statement is false
A) Occurs with inferior firearms and low velocity bullets
B) Produces large oval, triangular or cruciate entrance wound
C) Burning, blackening and tattooing are seen around the entrance wound
D) Abrasion collar is absent
Q33. A new serum marker that increases in a particular disease state is being evaluated. The
graph below depicts the relationship between sensitivity and false positive rate for different cut-
off values for the marker concentration in the serum.
Assuming that both correct diagnosis and ruling out the disease are equally important clinically,
which one of the values represents the appropriate cut-off value
A) 200
B) 5
C) 75
D) 50
Q34. Regarding Bupivacaine, a local anaesthetic
Assertion: It is cardiotoxic.
Reason: It blocks cardiac sodium channels and produces ventricular arrhythmias
ORIGINAL COPY
11 of 40
and myocardial depression.
A) Assertion and Reason are true and Reason is the correct explanation of assertion.
B) Assertion and Reason are true but Reason is not correct explanation of assertion
C) Assertion is true and reason is False.
D) Assertion is False and Reason is true.
Q35. The most common cause of acute pancreatitis is
A) Gall stones
B) Alcoholism
C) Malnutrition
D) Abdominal trauma
Q36. A low bioavailability of a drug may result from,
1. Failure of dosage form to disintegrate or dissolve in GI fluid
2. High first-pass metabolism
3. High active efflux transport of drug back into lumen.
4. High mucosal permeability.
A) Only 1, 2 and 3 are correct
B) Only 1 and 3 are correct
C) Only 2 and 4 are correct
D) Only 4 is correct.
Q37. Sarcoidosis associated hypercalcemia is treated with
A) Steroids
B) Thiazide diuretics
C) Bisphosphonates
D) Alkalinization of urine
Q38. The endolymph of inner ear is rich in
A) Potassium ions
B) Sodium ions
C) Chloride ions
D) Magnesium
Q39. Cessation of smoking for as short time as 12-48 hrs
A) Decreases carboxyhaemoglobin levels and nicotine levels
B) Decreases sputum production
ORIGINAL COPY
12 of 40
C) Improves pulmonary function tests
D) Normalizes immune function and metabolism
Q40. Which of the following is not a manifestation of Wilson disease?
A) Mitral valve stenosis
B) Hemolytic anemia
C) Cirrhosis
D) Dysarthria and dystonia
Q41. Regarding the process of callus formation following a fracture
A) Forms on periosteal surface only
B) Forms in response to some movement at fracture site
C) Lamellar bone is transformed into woven bone
D) Most often results in cartilage formation
Q42. A patient with bronchial asthma on bronchodilator therapy is scheduled for laparotomy.
The most suitable pulmonary function test to assess improvement in lung function is
A) Forced vital capacity
B) Total lung capacity
C) Peak expiratory flow rate
D) Maximum breathing capacity
Q43. Percentage of carbon-dioxide transported in carbaminohemoglobin form is
A) 7%
B) 23%
C) 70%
D) 35%
Q44. A patient with which of the following attributes is the most preferred patient for bariatric
surgery
A) Nonambulatory patient with BMI >40
B) BMI between 30-35 with no comorbidities
C) BMI>40 with no comorbidity
D) Depression due to morbid obesity
Q45. A 3 year male child presented with complaints of recurrent upper respiratory tract
infections, recurrent skin boils from the age of 6months. He was a product of a consanguineous
marriage. On examination he was malnourished, had nystagmus, hypopigmented hair, multiple
skin boils, generalized lymphadenopathy and mild hepatosplenomegaly. His elder sibling also
had similar complaints and died at the age of 5yrs. A blood examination done showed Hb of
ORIGINAL COPY
13 of 40
9gm%, TLC 15,000, Platelet 1.5 lakhs , normochromic, normocytic picture, large inclusions in
all the nucleated blood cells. The most probable diagnosis is
A) Chediak -Higashi syndrome
B) Chronic granulomatous disease
C) Hyper Ig E syndrome
D) Leukocyte adhesion deficiency
Q46. Footling presentation is a form of
A) Complete breech presentation
B) Frank breech presentation
C) Incomplete breech presentation
D) Compound presentation
Q47. In a child 2 months old with pertusis,
Assertion: The disease is only diagnosed after classical whoop develops
Reason: The catarrhal stage may last only few days
A) Both Assertion and Reason are true, and Reason is the correct explanation for
Assertion
B) Both Assertion and Reason are true, and Reason is not the correct explanation
for Assertion
C) Assertion is true, but Reason is false
D) Assertion is false, but Reason is true
Q48. Regarding relationship between ovarian and breast cancer,
Assertion: Women with ovarian cancer are at a higher risk for developing
breast cancer and vice-versa
Reason: Both malignancies have common oncogenic factors
A) Both Assertion and Reason are true, and Reason is the correct explanation for
Assertion
B) Both Assertion and Reason are true, and Reason is not the correct explanation
for Assertion
C) Assertion is true, but Reason is false
D) Assertion is false, but Reason is true
Q49. In a normal child, head circumference at the age of 2 years is approximately
A) 45 cms
B) 48 cms
C) 52 cms
D) 54 cms
ORIGINAL COPY
14 of 40
Q50. Following feature/features of indirect inguinal hernia differentiate it from direct inguinal
hernia
1. Neck of indirect hernia is lateral to epigastric vessels
2. Indirect hernia occurs in the elderly
3. Surgery of choice for indirect hernia in young adults is herniotomy
alone
4. Indirect hernia never strangulates
A) Only 1, 2 and 3 are correct
B) Only 1 and 3 are correct
C) Only 2 and 4 are correct
D) Only 4 is correct
Q51. Systemic inflammatory response syndrome (SIRS) is the body's systemic response to
severe infection. Incorrect statement regarding SIRS is
A) Lipopolysaccharide endotoxin from the walls of dying gram negative bacilli is
responsible
B) Interleukin-1 and tumour necrosis factor are mediators of inflammation
C) Leukopenia
D) Activated neutrophils adhere to vascular endothelium and decrease vascular
permeability
Q52. The rate constant of a biochemical reaction follows
A) Exponential increase with increase in temperature
B) Exponential decrease with increase in temperature
C) Linear increase with increase in temperature
D) Linear decrease with increase in temperature
Q53. An acute extradural hematoma is seen on CT as
A) Hyperdense concave lesion
B) Isodense lentiform (biconvex) lesion
C) Hyperdense lentiform (biconvex) lesion
D) Hypodense lentiform (biconvex) lesion
Q54. Regarding enteric fever
1. Complications such as gastrointestinal bleeding and intestinal perforation occur most
commonly in the 2nd
week of illness.
2. Rash (rose spots) in enteric fever occurs late in the course of the disease.
3. The definitive diagnosis of enteric fever rests on the demonstration of febrile
ORIGINAL COPY
15 of 40
agglutinins by Widal test
4. High dose fluoroquinolone therapy for 7 days for Nalidixic acid resistant (NAR)
enteric fever has been associated with delayed resolution of fever and high rates of
fecal carriage during convalescence.
A) Only 1, 2 and 3 are correct.
B) Only 1 and 3 are correct
C) Only 2 and 4 are correct
D) Only 4 is correct
Q55. The mutation due to substitution of a pyrimidine by a purine or vice versa is known as
A) Transition
B) Transversion
C) Replacement
D) Dynamic mutation
Q56. Regarding treatment of acute adrenal insufficiency
Assertion: glucocorticoids such as prednisolone are preferred
Reason: long acting glucocorticoids such as prednisolone result in
increased glucocorticoid exposure due to extended glucocorticoid
receptor activation
A) Both Assertion and Reason are true, and Reason is the correct explanation for
Assertion
B) Both Assertion and Reason are true, and Reason is not the correct explanation
for Assertion
C) Assertion is true, but Reason is false
D) Assertion is false, but Reason is true
Q57. Regarding Propofol which one is not correct
A) Has a pH of 7.0
B) Insoluble in water
C) Larger dosage is required in pediatric population
D) Propofol increases intracranial pressure
Q58. A 25-year old man, hailing from Eastern Uttar Pradesh, presented with fever, weight loss,
hepatosplenomegaly and pancytopenia. Bone marrow examination revealed the pathology as
depicted in the figure.
ORIGINAL COPY
16 of 40
A) Plasmodium falciparum
B) Candida
C) Leishmania donovani
D) Mycobacterium tuberculosis
Q59. Clinical features of Argyll Robertson pupil:
A) Light reflex is present
B) Seen in diabetes
C) Reaction to convergence and accommodation present
D) Pupil is dilated
Q60. During early pregnancy the hormone that mimics the action of LH is
A) Estrogen
B) Progesterone
C) HCG
D) Prolactin
Q61. In granulomatous inflammation due to tuberculosis, which of these statements is correct?
1 The granuloma consists of aggregates of epithelioid cells, lymphocytes and fibroblasts
2. Epithelioid cells fuse to form Langhans-type giant cells
3. There is central caseous necrosis
4. Numerous eosinophils are seen
A) Only 1, 2 & 3 are correct
B) Only 1 and 3 are correct
C) Only 2 and 4 are correct
D) Only 4 is correct
Q62. In spinal cord injury, bladder drainage technique with the lowest infection rate is
A) Clean intermittent catheterization
B) Supra-pubic drainage
C) Indwelling catheter
D) Condom catheter
ORIGINAL COPY
17 of 40
Q63. A 17 year old boy presented with crampy abdominal pain and a history of darkened stools
over the last few weeks. Physical examination was normal other than some vague central
abdominal tenderness and pigmentation of lower lip. An urgent ultrasound showed a probable
intussusception. Following reduction a follow up small bowel study showed multiple broad
based polyps mainly in the jejunum and ileum. What is the most likely diagnosis?
A) Familial adenomatous polyposis
B) Cowden syndrome
C) Juvenile polyposis
D) Peutz-Jeghers syndrome
Q64. Computed tomographic scan of the upper abdomen in a young girl who is complaining of
persistent upper abdominal pain has shown the following image
Which of the following statement holds true
A) Non vegetarian diet is responsible for this
B) Ultrasound guided needle aspiration cures the disease
C) Needs intervention only if it ruptures into the peritoneal cavity and causes
peritonitis
D) Laparotomy and pericystectomy are needed.
Q65. Brown-Sequard Syndrome consists of
A) Motor paralysis and loss of dorsal column sensations on the same side below the
lesion and loss of pain, temperature and crude touch on the opposite side.
B) Motor paralysis on the opposite side of the lesion.
C) Same as in A with loss of dorsal column sensations on the opposite side below
the lesion.
ORIGINAL COPY
18 of 40
D) Same as in A with no Motor Paralysis.
Q66. Cobra venom is mainly
A) Hepatotoxic
B) Neurotoxic
C) Ototoxic
D) Haematotoxic
Q67. The parotid salivary gland receives its parasympathetic innervation from which one of the
following nerves
A) Vagus
B) Facial
C) Glossopharyngeal
D) Oculomotor
Q68. Regarding tubercular arthritis
A) Primarily involves small joints of the hand
B) Systemic symptoms are seen in all patients
C) Usually associated with active pulmonary tuberculosis
D) Predominant cell type in synovial fluid is neutrophil
Q69. Regarding use of phototherapy in the management of neonatal hyperbilirubinemia:
1. Light in the blue range is most effective in reducing bilirubin levels
2. Ultraviolet radiation is preferred for phototherapy
3. Dark skin does not reduce the efficacy of phototherapy
4. Phototherapy helps convert unconjugated to conjugated bilirubin
A) Only 1, 2 and 3 are correct
B) Only 1 and 3 are correct
C) Only 2 and 4 are correct
D) All (1, 2, 3 and 4) are correct
Q70. Regarding Sydenham's chorea which is not true
A) ASO titres can be normal at the time of manifestation of chorea
B) Psychiatric disturbances can occur in the long term following chorea
C) Neurological deficits persist for upto 10 years
D) Reported incidence 5-35% in rheumatic fever
ORIGINAL COPY
19 of 40
Q71. Regarding p value
Assertion: A p value of 0.05 is more significant than 0.01
Reason: It gives an estimate of the probability of a result occurring by
chance.
A) Both Assertion and Reason are true and Reason is the correct explanation for
Assertion
B) Both Assertion and Reason are true and Reason is not the correct explanation for
Assertion
C) Assertion is true but Reason is false
D) Reason is true but Assertion is false
Q72. 'My head is missing'; 'I have no body'; 'I am dead'- these are symptoms described in
A) Hypochondriasis
B) Persecutory Delusion
C) Grandiose Delusion
D) Nihilistic Delusion
Q73. Diagnostic biochemical investigation in Paget's disease is by estimating
A) 24 hour urinary calcium excretion
B) 24 hour urinary Phosphorous excretion
C) Urinary alkaline phosphatase
D) 24 hour urinary hydroxy proline
Q74. Korner's Septum is present in
A) Petrosquamous suture
B) Tympanomastoid suture
C) Tympano Squamous suture
D) Tympano petrous petrous suture
Q75. Pre ganglionic para sympathetic fibres to otic ganglion are carried by:
1. Auriculo-temporal nerve.
2. Temporal branch of facial nerve.
3. Auricular branch of vagus nerve.
4. Lesser petrosal nerve.
ORIGINAL COPY
20 of 40
A) Only 1, 2 & 3 are correct.
B) Only 1 and 3 are correct.
C) Only 2 & 4 are correct.
D) Only 4 is correct.
Q76. Regarding proton pump inhibitors:
Assertion: Proton pump inhibitors should ideally be given 30 minutes before meals.
Reason: Acidic pH in gastric lumen increases absorption of proton pump inhibitors.
A) Both Assertion and Reason are true, and Reason is the correct explanation for
Assertion
B) Both Assertion and Reason are true, and Reason is not the correct explanation
for Assertion
C) Assertion is true, but Reason is false
D) Assertion is false, but Reason is true
Q77. In severe diarrhea, dehydration and electrolyte imbalance pose principal risk especially in
children.
Assertion: Oral Rehydration therapy is cornerstone for patients with acute illnesses resulting in
significant diarrhea.
Reason: Nutrient linked cotransport of water and electrolytes remains intact in small bowel in
most cases of acute diarrhea.
A) Both Assertion and Reason are true, and Reason is the correct explanation for
Assertion
B) Both Assertion and Reason are true, and Reason is not the correct explanation
for Assertion
C) Assertion is true, but Reason is false
D) Assertion is false, but Reason is true
Q78. A 39 year old was imaged following sudden onset right hemiplegia. A CT showed subtle
atrophy of the left occipital lobe and unusual gyral calcification. MRI demonstrated prominent
pial and deep medullary veins in the region with a prominent left choroid plexus. What is the
underlying aetiology?
A) Tuberous sclerosis
ORIGINAL COPY
21 of 40
B) Ataxia telangiectasia
C) Von Hippel-Lindau syndrome
D) Sturge-Weber syndrome
Q79. A 6 week old infant is admitted to the hospital with jaundice. On investigating for jaundice,
he has total serum bilirubin of 13 mg/dl and direct fraction of 2 mg/dl.
Condition/conditions which can be suspected
1. Choledochal cyst
2. Gilbert Syndrome
3. Galactosemia
4. Crigler - Najjar Syndrome
A) Only 1, 2 and 3 are correct
B) Only 1 and 3 are correct
C) Only 2 and 4 are correct
D) Only 4 is correct
Q80. Regarding congenital defect in the development of face,
Assertion: Oblique facial cleft extends from the medial angle of eye to mouth (upper lip)
Reason: Maxillary process does not fuse with the palatine process
A) Both Assertion and Reason are true, and Reason is the correct explanation for
Assertion
B) Both Assertion and Reason are true, and Reason is not the correct explanation
for Assertion
C) Assertion is true, but Reason is false
D) Assertion is false, but Reason is true
Q81. Regarding anticholinergics in bronchial asthma
Assertion: They are equally effective as Beta2 agonists in asthma therapy
Reason: They act by inhibiting only the cholinergic reflex component of
bronchoconstriction.
ORIGINAL COPY
22 of 40
A) Both Assertion and Reason are true, and Reason is the correct explanation for
the Assertion
B) Both Assertion and Reason are true, and Reason is not the correct
Explanation for Assertion
C) Assertion is true, but Reason is false
D) Assertion is false, but Reason is true
Q82. In humans if both noradrenergic and cholinergic systems are blocked, the approximate
heart rate would be approximately
A) 70
B) 50
C) 100
D) 120
Q83. Left atrial enlargement can be recognized on a chest radiograph as
1. Rounding of left heart border
2. Double contouring of right cardiac shadow
3. Elevated cardiac apex
4. Widening of carina
A) Only 1, 2 and 3 are correct
B) Only 1 and 3 are correct
C) Only 2 and 4 are correct
D) Only 4 is correct
Q84. Among the Duke criteria for the clinical diagnosis of infective endocarditis, the major
criteria include
1. Growth of a typical microorganism for Infective endocarditis from two separate blood
cultures.
2. Osler's nodes
ORIGINAL COPY
23 of 40
3. Single positive blood culture for Coxiella burnetii.
4. Fever ≥ 38.00
C(≥ 100.40
F)
A) Only 1, 2 and 3 are correct.
B) Only 1 and 3 are correct
C) Only 2 and 4 are correct
D) Only 4 is correct
Q85. Which of the following is not a mechanism of action of Mifepristone (RU-486)
A) Competes with progesterone receptors
B) Causes sloughing of endometrium
C) Causes uterine contraction and softening of cervix
D) Has Glucocorticoid and Androgenic action.
Q86. Regarding use of tamoxifen in patients of ER positive metastatic breast cancer
Assertion: Patients with CYP2D6 polymorphisms that reduce its activity, are associated
with higher risk of disease relapse.
Reason: Tamoxifen is metabolized in liver by CYP2D6 into inactive metabolites, 4-OH
Tamoxifen and endoxifen
A) Both Assertion and Reason are true, and Reason is the correct explanation for
Assertion
B) Both Assertion and Reason are true, and Reason is not the correct explanation
for Assertion
C) Assertion is true, but Reason is false
D) Assertion is false, but Reason is true
Q87. Which of the following is not a risk factor for Candidial Vaginitis
A) Prolonged steroid therapy
B) Type II diabetes mellitus
C) Thin statured woman
D) Long term antibiotic therapy
Q88. Identify the abnormality in the following chest radiograph
ORIGINAL COPY
24 of 40
A) Fracture of left clavicle
B) Fracture of 3rd rib Right side
C) Bilateral cervical ribs
D) Anterior Mediastinal Mass
Q89. Which of the following is a component of the massive transfusion resuscitation protocol
used to improve survival in the setting of severe trauma
A) Use of large volume of crystalloids
B) Use of whole blood for immediate resuscitation
C) Use of unmatched O negative blood for immediate resuscitation
D) For patients requiring more than 2 units of packed red blood cells 2 units of FFP
should be given
Q90. A 19 year old girl comes to out-patient with Primary amenorrhea. On examination, she has
well developed Secondary sexual characters. FSH, LH levels are found to be normal and
karyotyping is 46 XX. Ultrasound shows absent uterus and vagina. The diagnosis is
A) Turner syndrome
B) Savage syndrome.
C) Rokitansky-Kustner Hauser syndrome.
D) Kallmans syndrome.
Q91. Regarding advice of following a traditional low fat diet
Assertion: It is a primordial preventive measure for coronary artery disease
ORIGINAL COPY
25 of 40
Reason: It reduces the risk of conversion of angina to myocardial infarction
A) Both Assertion and Reason are true, and Reason is the correct explanation for
Assertion
B) Both Assertion and Reason are true, and Reason is not the correct explanation for
Assertion
C) Assertion is true, but Reason is false
D) Assertion is false, but Reason is true
Q92. A 50 year old man with history of persistent shoulder pain had a jerking injury of the
shoulder. On examination, 6 weeks later, he is unable to abduct his arm. Passive abduction was
normal and he could keep the arm up once it had been raised above the right angle. This suggests
A) Partial tear of rotator cuff
B) Full thickness tear of rotator cuff
C) Isolated infraspinatus weakness
D) Isolated subscapularis weakness
Q93. Second degree inversion of uterus is
A) Uterine fundus just inverts into the uterine cavity
B) Uterine fundus is at the cervix
C) Uterine fundus protrudes through cervix and lies in vagina
D) Whole of uterus is inverted and protrudes through the introitus
Q94. Which of the following anthropometric parameters is not used as an indicator of the
nutritional status of a toddler?
A) Mid-upper arm circumference
B) Weight for height
C) Occipito-frontal head circumference
D) Weight for age
Q95. Regarding breast self examination
Assertion: Monthly breast self examination is recommended by American Cancer
Society
ORIGINAL COPY
26 of 40
Reason: Breast self examination is as sensitive as mammography
A) Both Assertion and Reason are true, and reason is the correct explanation for
assertion
B) Both Assertion and reason are true, and Reason is not the correct explanation
for assertion
C) Assertion is true, but Reason is false
D) Assertion is false, but Reason is true
Q96. What is true in the given below data of weights of 9 school children chosen randomly.
20, 31, 31, 31, 25, 28, 35, 38, 31.
A) Mean is 31
B) Range is 20-38
C) Median is 15
D) Mode is 15
Q97. In a fire-arm injury ' shored exit' is the one, where
A) Exit wound is slightly smaller than the entry wound.
B) Exit wound is larger than the entry wound.
C) Both Exit and entry wounds edges are inverted
D) Both entry and exit wounds have abrasion collar at the margins.
Q98. The mechanism of whip-lash injury to cervical spine is essentially
A) Sudden extension of the neck followed by flexion.
B) Sudden flexion followed by extension
C) Sudden distraction
D) Axial loading
Q99. Juxta glomerular apparatus consists of
ORIGINAL COPY
27 of 40
A) Only 1, 2 & 3 are correct
B) Only 1 & 3 are correct
C) Only 2 & 4 are correct
D) Only 4 correct
Q100. Which of the following interstitial lung diseases is not associated with smoking?
A) Desquamative interstitial pneumonitis
B) Bronchiolitis obliterans organizing pneumonia
C) Pulmonary Langerhans cell granulomatosis
D) Respiratory bronchiolitis-interstitial lung disease
Q101. Unilateral dysmenorrhea occurs in
A) Fibroid uterus
B) Rudimentary horn of bicornuate uterus
C) Endometriotic Cyst
D) Pelvic inflammatory disease,
Q102. The cross-section of the original Kuntscher nail (K- Nail) designed for the femur
is
A) Triangular
B) Circular
C) Oval
D) Clover leaf
Q103. The following are indications for strongly considering cardiac surgical
ORIGINAL COPY
28 of 40
intervention in patients with endocarditis.
1. Perivalvular extension of infection
2. Poorly responsive or relapsed endocarditis due to highly antibiotic-resistant
enterococci or gram negative bacilli
3. Persistent unexplained fever(≥ 10days) in culture negative native valve endocarditis.
4. Vegetation of 5 mm in diameter.
A) Only 1, 2 and 3 are correct.
B) Only 1 and 3 are correct
C) Only 2 and 4 are correct
D) Only 4 is correct
Q104. A young patient presents with a massive injury to proximal duodenum, head of pancreas
and distal common bile dyct. He is hemodyanamically stable. The surgical procedure of choice
in this patient should be
A) Roux - en-Y anastomosis
B) Pancreaticoduodenectomy (Whipple's operation)
C) Lateral tube jejunostomy
D) Retrograde jejunostomy
Q105. True statement regarding Frey's syndrome is
A) Rare complication after parotidectomy
B) Results from damage to autonomic innervation of the salivary gland with
inappropriate regeneration of sympathetic nerve fibres that stimulate the sweat
glands of the overlying skin
C) Revision surgery and excision of affected skin and placement of skin graft is
definite treatment
D) Established Frey's syndrome can best be addressed by injection of botulinum
toxin into the affected skin
Q106. Identify the tissue given below
ORIGINAL COPY
29 of 40
A) Dermis
B) Epidermis
C) Hair follicle
D) Sebaceous gland
Q107. Methylated purines and pyrimidines are characteristically present in
A) mRNA
B) hnRNA
C) tRNA
D) rRNA
Q108. Mean corpuscular hemoglobin concentration (MCHC) is increased in following
condition
A) Hereditary spherocytosis
B) Iron deficiency anemia
C) Thalassemia
D) Sickle cell diseases
Q109. Which of the following is not a part of the Rotator Cuff?
A) Supra Spinatus
B) Teres Major
C) Infra Spinatus
D) Sub Scapularis
Q110. Aminoglycoside antibiotics are
ORIGINAL COPY
30 of 40
A) Bacteriostatic, effective against aerobic gram positive bacilli
B) Bacteriostatic, effective against aerobic gram negative bacilli
C) Bactericidal, effective against aerobic gram positive bacilli
D) Bactericidal, effective against aerobic gram negative bacilli
Q111. Regarding the laboratory diagnosis of M. tuberculosis infection
1. For patients with suspected pulmonary Tuberculosis, it is recommended that two or three
sputum specimens collected early in the morning should be submitted for AFB smear and
Mycobacterial culture.
2. Serologic tests based on detection of antibodies to a variety of mycobacterial antigens are not
useful for the diagnosis of tuberculosis.
3. Tuberculin skin testing is of limited value in the diagnosis of active TB.
4. Fluorescence microscopy of specimens stained with auramine -rhodamine has the same
sensitivity as light microscopy of specimens stained with Ziehl-Neelsen stain.
A) Only 1, 2 and 3 are correct.
B) Only 1 and 3 are correct
C) Only 2 and 4 are correct
D) Only 4 is correct
Q112. An infant was vaccinated with measles vaccine at the age of 6 months due a measles
outbreak in the village.
Assertion: A second dose of measles vaccine is recommended as soon as
possible after 9 months of age
Reason: A different strain of virus can cause measles after an outbreak.
A) Both Assertion and Reason are true and Reason is the correct explanation for
Assertion
B) Both Assertion and Reason are true and Reason is not the correct explanation for
Assertion
C) Assertion is true but Reason is false
D) Reason is true but Assertion is false
Q113. What chemical is used in identifying fungal infection of skin
A) 20% potassium hydroxide
B) Cyano acrylate
C) Para amino benzoic acid
D) 20% sulphuric acid
Q114. Given ECG leads suggest
ORIGINAL COPY
31 of 40
A) Left atrial enlargement
B) Right ventricular hypertrophy
C) Left ventricular enlargement
D) Both right and left atrial enlargement
Q115. Index/indices used in calculating human development index
1) Life expectancy at birth
2) Adult literacy rate
3) Gross domestic product (GDP) per capita
4) Maternal mortality rate
A) Only 1, 2 and 3 are correct
B) Only 1 and 3 are correct
C) Only 2 and 4 are correct
D) Only 4 is correct
Q116. The diplopia chart in patient complaining of diplopia is given below
Patients R Centre Patients L
↑ ↑ ↑↑ ↑
↑ ↑ ↑↑ ↑
↑ ↑ ↑↑ ↑
The most likely cause is
A) Right lateral rectus palsy
B) Right medial rectus palsy
C) Left trochlear nerve lesion
D) Right occulomotor nerve lesion
Q117. A 25 years old lady comes to the hospital with history of 2 second trimester abortions.
Both times she describes the events as being painless with expulsion of liquor first followed by
ORIGINAL COPY
32 of 40
the immature fetus. The most probable cause of fetal loss is
A) Congenital abnormalities
B) Intrauterine growth Retardation (IUGR)
C) Infection
D) Cervical incompetence
Q118. The only abductor of the vocal cord is
A) Cricothyroid
B) Vocalis
C) Transverse arytenoid
D) Posterior cricoarytenoid
Q119. Layers of retina include/includes
1) Nerve fibre layer
2) Internal limiting membrane
3) Retinal pigment epithelium
4) Bowman's membrane
A) Only 1, 2 and 3 are correct
B) Only 1 and 3 are correct
C) Only 2 and 4 are correct
D) Only 4 is correct
Q120. Submucosal glands are absent in
A) Duodenum
B) Oesophagus
C) Trachea
D) Appendix
Q121. Which of the following is a structural analog of tyrosinyl-tRNA?
A) Puromycin
B) Streptomycin
C) Actinomycin D
D) Cyclohexamide
Q122. The following drugs have activity against hepatitis B virus
ORIGINAL COPY
33 of 40
1. Lamivudine
2. Emtricitabine
3. Tenofovir
4. Efavirenz
A) Only 1, 2 and 3 are correct
B) Only 1and 3 are correct
C) Only 2 and 4 are correct
D) Only 4 is correct
Q123. A screening test for cancer has been evaluated in 400 people and results are shown below
in table
Screening
test
Cancer No cancer Total
Positive 90 30 120
Negative 10 270 280
Total 100 300 400
The positive predictive value for the screening test is
A) 90%
B) 10%
C) 75%
D) 96%
Q124. A 34-year-old man with HIV/AIDS (CD4+ T-lymphocyte count = 110/mm3
) develops a
scaly, waxy, yellowish, patchy, crusty, pruritic rash on and around his nose. The rest of his skin
examination is normal. Which of the following is the most likely diagnosis?
A) Molluscum contagiosum
B) Seborrheic dermatitis
C) Reactivation herpes zoster
D) Psoriasis
Q125. "Russell's" sign is seen in
A) Anorexia Nervosa
B) Bulimia Nervosa
C) Somatization disorder
D) Panic Disorder
Q126. Electrolyte imbalance has to be regularly corrected in intensive care unit patients.
Approximate daily requirement of potassium is
ORIGINAL COPY
34 of 40
A) 10 milli moles
B) 30 milli moles
C) 50 milli moles
D) 70 milli moles
Q127. The Ultrasound of a G2 P1L1 done at 26 weeks, showed a "Double Bubble" sign in the
fetus. The most probable diagnosis is
A) Gastroschisis
B) Omphalocoel
C) Duodenal Atresia
D) Polycystic Kidney
Q128. True statement regarding nephrogenic fibrosing dermopathy associated with magnetic
resonance contrast agent gadolinium
A) Patient with stage 2 chronic kidney disease can be administered normal amount
of gadolinium
B) Associated facial fibrosis is a rule
C) Concurrent liver disease is not a risk factor
D) Gadolinium can be removed rapidly by hemodialysis
Q129. In cases of closed lower limb injuries, it is always important to suspect compartment
syndrome and to take preventive measures. Which of the following statement is wrong regarding
compartment syndrome?
A) Characterized by severe pain, distal sensory disturbance and absence of distal
pulses.
B) When in doubt of this situation, it is not safe to do fasciotomy
C) Delayed fasciotomy may cause myoglobinuria and renal failure
D) It can even occur in open injuries if the wound doesn't extend into the affected
compartment.
Q130. In the case of the ureters, the anatomical points of constriction are
1. At the ureteropelvic junction, just inferior to the kidney.
ORIGINAL COPY
35 of 40
2. Where ureters cross the common iliac vessels at the pelvic brim.
3. Where ureters enter the wall of bladder.
4. At the renal sinuses.
A) 1, 2, 3 are correct
B) Only 1, 3 are correct
C) Only 2, 4 are correct
D) Only 4 is correct
Q131. A small change in volume leads to an inordinate increase in intracranial pressure when
A) Intracranial compliance is high
B) Intracranial elastance is high
C) Intracranial elastance is low
D) Intracranial resistance is low
Q132.A man with hemophilia A married a normal woman whose father had
Hemophilia A. Regarding their children
Assertion: 50% of female children will suffer from the disease
Reason: The abnormality is inherited as X linked dominant
A) Both Assertion and Reason are true, and Reason is the correct explanation for
Assertion
B) Both Assertion and Reason are true, and Reason is not the correct explanation for
Assertion
C) Assertion is true, but Reason is false
D) Assertion is false, but Reason is true
Q133. Regarding Modified Ritgens manoeuvre,
Assertion: It allows controlled delivery of the head
Reason: This position favours neck extension so that the head is delivered by the smallest
diameter passing through the introitus
A) Both Reason and Assertion are true and Reason is the correct explanation for
Assertion
B) Both Assertion and Reason are true and Reason is not the correct explanation for
the Assertion
C) Assertion is true but Reason is false
D) Assertion is false but Reason is true.
Q134. In the process of secondary active transport involved in molecule movement
across cell membranes
A) Energy expenditure is minimal
ORIGINAL COPY
36 of 40
B) Transport of Na + is coupled to that of other molecules
C) Osmotic gradient is established across cells
D) Size of intercellular junction is a limiting factor
Q135. Jerger's curve on impedance audiometry in post stapedotomy case will be
A) B Type
B) Ad type
C) As type
D) C type
Q136. A 45 year old lady, a hypertensive came with complaints of partial visual deficits and her
fundus picture is given below
The most likely diagnosis is
A) Papillitis
B) Branched retinal vein occlusion
C) Retinitis pigmentosa
D) Central retinal artery occlusion
Q137. Which of the following drug acts by inhibiting the enzyme Inosine
monophosphate dehydrogenase
A) Azathioprine
B) Alefacept
C) Mycophenolate mofetil
D) Pimecrolimus
Q138. PCOD (Poly Cystic Ovarian Disease) in earlier life may be attributed to which of the
following in later life
1. Hypertension
ORIGINAL COPY
37 of 40
2. Type 2 diabetes mellitus
3. Endometrial carcinoma
4. Ovarian carcinoma
A) Only 1, 2 and 3 are correct.
B) Only 1, 3 are correct
C) Only 2, 4 are correct
D) Only 4 is correct
Q139. Jefferson fracture is-
A) Fracture of atlas ring caused by flexion
B) Fracture of atlas ring caused by axial loading
C) Fracture of axis ring caused by flexion
D) Fracture of axis ring caused by axial loading
Q140. A 12-year old child had a sore-throat, following which, he presented with fever, malaise,
nausea, oliguria and smoky urine. BP was 130/90 mm of Hg. Urine examination revealed
proteinuria (750 mg/day) and red cell casts. The renal biopsy revealed hypercellular glomeruli
infiltrated by neutrophils with proliferation of endothelial cells and mesangial cells. The likely
diagnosis would be
A) Minimal-change disease
B) Post-infectious glomerulonephritis
C) Alport syndrome
D) Good Pasture syndrome
Q141. At autopsy, massive splenomegaly was detected, with gross appearance of "sago spleen".
Microscopic sections reveal deposition of amorphous, eosinophilic material, limited to the
splenic follicles .The likely diagnosis would be
A) Miliary tuberculosis
B) Amyloidosis
C) Septic abscesses
D) Non-Hodgkins lymphoma
Q142. Bouchard's Nodes are seen in
A) Meta - Carpo Phalangeal joints
ORIGINAL COPY
38 of 40
B) Distal inter- Phalangeal joints
C) Proximal Inter- Phalangeal joints
D) Carpo - Meta carpal joints.
Q143. Regarding ESBL producing strains of Gram negative bacilli,
Assertion: Are resistant to multiple antibiotic groups and may include fluoroquinolones,
aminoglycosides, beta lactamase inhibitor combinations and carbapenems.
Reason: A single determinant on a resistance plasmid codes for resistance to all the
groups of antibiotics.
A) Both Assertion and Reason are true, and Reason is the correct explanation for
Assertion
B) Both Assertion and Reason are true, and Reason is not the correct explanation
for Assertion
C) Assertion is true, but Reason is false
D) Assertion is false, but Reason is true
Q144. Which of the following hormone uses phospholipase C as second messenger
A) Secretin
B) Oxytocin
C) Calcitonin
D) Somatostatin
Q145. Calot's triangle or hepatobiliary triangle is an important landmark for hepatobiliary
surgeries. Its boundaries are formed by:
A) Cystic duct, right hepatic duct and common hepatic duct
B) Common bile duct, cystic duct and cystic artery
C) Cystic duct, common bile duct and duodenum
D) Cystic duct, cystic artery and common hepatic artery
Q146. The virus known to be associated with nasopharyngeal carcinoma is
A) Cytomegalovirus
B) Hepatitis B virus
C) Hepatitis C virus
D) Epstein Barr virus
Q147. Which of the following terms is not used to describe pathology in rheumatic heart
disease?
ORIGINAL COPY
39 of 40
A) Rokitansky-Aschoff sinuses
B) Aschoff bodies
C) Anitschkow cells
D) Mac Callum plaques
Q148. Shown below is the distribution of the laboratory test results in normal and two
different disease states (disease 1 and disease 2).
Assertion: The laboratory test is more reliable to diagnose disease 1 than the disease 2
Reason: Test reliability decreases if test result values in diseased population overlap with those
of the normal population
A) Assertion and Reason are correct and Reason is the correct explanation of the
Assertion
B) Assertion and Reason are correct and Reason is not the correct explanation of
the Assertion
C) Assertion is true and Reason is false
D) Assertion is false and Reason is true
Q149. Key features of biologic agents used as bioweapons are
1. High morbidity and mortality rates.
2. High infective dose
3. Highly infectious by aerosol.
4. No potential for person to person spread
A) Only 1, 2 and 3 are correct.
B) Only 1 and 3 are correct
C) Only 2 and 4 are correct
D) Only 4 is correct
Q150. Cell of origin of meningioma is
A) Oligodendrocyte
ORIGINAL COPY
40 of 40
B) Arachnoid cap cell
C) Neuroblast
D) Pial cell

More Related Content

What's hot

Dnb pediatrics osce 2 for PGS in Southern Railway Hospital
Dnb pediatrics osce 2 for PGS in Southern Railway HospitalDnb pediatrics osce 2 for PGS in Southern Railway Hospital
Dnb pediatrics osce 2 for PGS in Southern Railway HospitalNibedita Mitra
 
OSCE in Pediatrics (Wadia, Sept 2011)
OSCE in Pediatrics (Wadia, Sept 2011)OSCE in Pediatrics (Wadia, Sept 2011)
OSCE in Pediatrics (Wadia, Sept 2011)Dr Padmesh Vadakepat
 
OSCE Pediatrics Dr.Mehta Hospital 2012
OSCE Pediatrics Dr.Mehta Hospital 2012OSCE Pediatrics Dr.Mehta Hospital 2012
OSCE Pediatrics Dr.Mehta Hospital 2012Dr Padmesh Vadakepat
 
Toacs imm january.2015
Toacs imm january.2015Toacs imm january.2015
Toacs imm january.2015zahid mehmood
 
Pediatric Arab Board MCQ Review - Emergency Medicine
Pediatric Arab Board MCQ Review - Emergency Medicine Pediatric Arab Board MCQ Review - Emergency Medicine
Pediatric Arab Board MCQ Review - Emergency Medicine Fatima Farid
 
Anesthesia complications mcqs by israr ysfz
Anesthesia complications mcqs by israr ysfzAnesthesia complications mcqs by israr ysfz
Anesthesia complications mcqs by israr ysfzisrar khan
 
Expression of Isocitrate Dehydrogenase-1 (IDH-1) Mutant Protein in Gliomas_Cr...
Expression of Isocitrate Dehydrogenase-1 (IDH-1) Mutant Protein in Gliomas_Cr...Expression of Isocitrate Dehydrogenase-1 (IDH-1) Mutant Protein in Gliomas_Cr...
Expression of Isocitrate Dehydrogenase-1 (IDH-1) Mutant Protein in Gliomas_Cr...CrimsonPublishersTNN
 
Acyclovir Induced Acute Kidney Injury In Acute Meningitis Patient: A Case Rep...
Acyclovir Induced Acute Kidney Injury In Acute Meningitis Patient: A Case Rep...Acyclovir Induced Acute Kidney Injury In Acute Meningitis Patient: A Case Rep...
Acyclovir Induced Acute Kidney Injury In Acute Meningitis Patient: A Case Rep...iosrphr_editor
 
STEP 2 - Nums past paper by dr. shahid alam
STEP 2 - Nums past paper by dr. shahid alamSTEP 2 - Nums past paper by dr. shahid alam
STEP 2 - Nums past paper by dr. shahid alamDr. Shadab Kamal
 

What's hot (20)

OSCE Pediatrics KKCTH
OSCE Pediatrics KKCTHOSCE Pediatrics KKCTH
OSCE Pediatrics KKCTH
 
Senior Medillectuals Prelims
Senior Medillectuals Prelims Senior Medillectuals Prelims
Senior Medillectuals Prelims
 
Dnb pediatrics osce 2 for PGS in Southern Railway Hospital
Dnb pediatrics osce 2 for PGS in Southern Railway HospitalDnb pediatrics osce 2 for PGS in Southern Railway Hospital
Dnb pediatrics osce 2 for PGS in Southern Railway Hospital
 
Vars
VarsVars
Vars
 
Mock OSCE Pediatrics Apr 2013
Mock OSCE Pediatrics Apr 2013Mock OSCE Pediatrics Apr 2013
Mock OSCE Pediatrics Apr 2013
 
Clinical cases (1) june 16, 13
Clinical cases  (1) june 16, 13Clinical cases  (1) june 16, 13
Clinical cases (1) june 16, 13
 
DNB Pediatrics OSCE June 2013
DNB Pediatrics OSCE June 2013DNB Pediatrics OSCE June 2013
DNB Pediatrics OSCE June 2013
 
OSCE in Pediatrics (Wadia, Sept 2011)
OSCE in Pediatrics (Wadia, Sept 2011)OSCE in Pediatrics (Wadia, Sept 2011)
OSCE in Pediatrics (Wadia, Sept 2011)
 
OSCE Pediatrics Dr.Mehta Hospital 2012
OSCE Pediatrics Dr.Mehta Hospital 2012OSCE Pediatrics Dr.Mehta Hospital 2012
OSCE Pediatrics Dr.Mehta Hospital 2012
 
Toacs imm january.2015
Toacs imm january.2015Toacs imm january.2015
Toacs imm january.2015
 
Pediatric Arab Board MCQ Review - Emergency Medicine
Pediatric Arab Board MCQ Review - Emergency Medicine Pediatric Arab Board MCQ Review - Emergency Medicine
Pediatric Arab Board MCQ Review - Emergency Medicine
 
Anesthesia complications mcqs by israr ysfz
Anesthesia complications mcqs by israr ysfzAnesthesia complications mcqs by israr ysfz
Anesthesia complications mcqs by israr ysfz
 
RIPE 2012 Pediatrics OSCE
RIPE 2012 Pediatrics OSCERIPE 2012 Pediatrics OSCE
RIPE 2012 Pediatrics OSCE
 
OSCE Pediatrics
OSCE PediatricsOSCE Pediatrics
OSCE Pediatrics
 
O d
O dO d
O d
 
Pediatrics OSCE, RIPE 2013
Pediatrics OSCE, RIPE 2013 Pediatrics OSCE, RIPE 2013
Pediatrics OSCE, RIPE 2013
 
Pediatrics OSCE
Pediatrics OSCEPediatrics OSCE
Pediatrics OSCE
 
Expression of Isocitrate Dehydrogenase-1 (IDH-1) Mutant Protein in Gliomas_Cr...
Expression of Isocitrate Dehydrogenase-1 (IDH-1) Mutant Protein in Gliomas_Cr...Expression of Isocitrate Dehydrogenase-1 (IDH-1) Mutant Protein in Gliomas_Cr...
Expression of Isocitrate Dehydrogenase-1 (IDH-1) Mutant Protein in Gliomas_Cr...
 
Acyclovir Induced Acute Kidney Injury In Acute Meningitis Patient: A Case Rep...
Acyclovir Induced Acute Kidney Injury In Acute Meningitis Patient: A Case Rep...Acyclovir Induced Acute Kidney Injury In Acute Meningitis Patient: A Case Rep...
Acyclovir Induced Acute Kidney Injury In Acute Meningitis Patient: A Case Rep...
 
STEP 2 - Nums past paper by dr. shahid alam
STEP 2 - Nums past paper by dr. shahid alamSTEP 2 - Nums past paper by dr. shahid alam
STEP 2 - Nums past paper by dr. shahid alam
 

Similar to NIMSET 2013 Questions

Golden book for Medicine OSCE: First View
Golden book for Medicine OSCE: First ViewGolden book for Medicine OSCE: First View
Golden book for Medicine OSCE: First ViewMan B Paudyal
 
Krok 2 - 2013 Question Paper (General Medicine)
Krok 2 - 2013 Question Paper (General Medicine)Krok 2 - 2013 Question Paper (General Medicine)
Krok 2 - 2013 Question Paper (General Medicine)Eneutron
 
Questions 1700 - Medical MCQ without Answer
Questions 1700 - Medical MCQ without AnswerQuestions 1700 - Medical MCQ without Answer
Questions 1700 - Medical MCQ without AnswerMBBS Help
 
Step 1 PMDC paper compiled by Dr. Asif ali
Step 1 PMDC paper compiled by Dr. Asif aliStep 1 PMDC paper compiled by Dr. Asif ali
Step 1 PMDC paper compiled by Dr. Asif aliDr. Shadab Kamal
 
AIIMS 2001 quetions
AIIMS 2001 quetionsAIIMS 2001 quetions
AIIMS 2001 quetionsmedicospace
 
Dermatology MCQ and AAFP.pptx
Dermatology MCQ and AAFP.pptxDermatology MCQ and AAFP.pptx
Dermatology MCQ and AAFP.pptxAbdulaziz Bagasi
 
Pneumoniatutor 180316220436
Pneumoniatutor 180316220436Pneumoniatutor 180316220436
Pneumoniatutor 180316220436JohnCooper855841
 
COUNCIL THEORY 2018 ANSWERED-1.docx
COUNCIL THEORY 2018 ANSWERED-1.docxCOUNCIL THEORY 2018 ANSWERED-1.docx
COUNCIL THEORY 2018 ANSWERED-1.docxRBGroup
 
15 soal basic science
15 soal basic science15 soal basic science
15 soal basic scienceRizwandhaAzmi
 
Review course 2013 answer key.anand.tiwari
Review course 2013 answer key.anand.tiwariReview course 2013 answer key.anand.tiwari
Review course 2013 answer key.anand.tiwariAnand Tiwari
 
IVMS Comprehensive -USMLE-type- Pharmacology-#1-2012 Examination
IVMS Comprehensive -USMLE-type- Pharmacology-#1-2012 ExaminationIVMS Comprehensive -USMLE-type- Pharmacology-#1-2012 Examination
IVMS Comprehensive -USMLE-type- Pharmacology-#1-2012 ExaminationImhotep Virtual Medical School
 

Similar to NIMSET 2013 Questions (20)

Segundo simulador primera parte
Segundo simulador primera parteSegundo simulador primera parte
Segundo simulador primera parte
 
Golden book for Medicine OSCE: First View
Golden book for Medicine OSCE: First ViewGolden book for Medicine OSCE: First View
Golden book for Medicine OSCE: First View
 
Krok 2 - 2013 Question Paper (General Medicine)
Krok 2 - 2013 Question Paper (General Medicine)Krok 2 - 2013 Question Paper (General Medicine)
Krok 2 - 2013 Question Paper (General Medicine)
 
Questions 1700 - Medical MCQ without Answer
Questions 1700 - Medical MCQ without AnswerQuestions 1700 - Medical MCQ without Answer
Questions 1700 - Medical MCQ without Answer
 
Senior Medillectuals- Mains
Senior Medillectuals- MainsSenior Medillectuals- Mains
Senior Medillectuals- Mains
 
Step 1 PMDC paper compiled by Dr. Asif ali
Step 1 PMDC paper compiled by Dr. Asif aliStep 1 PMDC paper compiled by Dr. Asif ali
Step 1 PMDC paper compiled by Dr. Asif ali
 
AIIMS 2001 quetions
AIIMS 2001 quetionsAIIMS 2001 quetions
AIIMS 2001 quetions
 
Dermatology MCQ and AAFP.pptx
Dermatology MCQ and AAFP.pptxDermatology MCQ and AAFP.pptx
Dermatology MCQ and AAFP.pptx
 
Mmv sample-mc qs (1)
Mmv sample-mc qs (1)Mmv sample-mc qs (1)
Mmv sample-mc qs (1)
 
Pneumoniatutor 180316220436
Pneumoniatutor 180316220436Pneumoniatutor 180316220436
Pneumoniatutor 180316220436
 
Clinical cases march 24,13
Clinical cases march 24,13Clinical cases march 24,13
Clinical cases march 24,13
 
Clinical cases (1) june 30, 13
Clinical cases  (1) june 30, 13Clinical cases  (1) june 30, 13
Clinical cases (1) june 30, 13
 
Clinical cases march 24,13
Clinical cases march 24,13Clinical cases march 24,13
Clinical cases march 24,13
 
COUNCIL THEORY 2018 ANSWERED-1.docx
COUNCIL THEORY 2018 ANSWERED-1.docxCOUNCIL THEORY 2018 ANSWERED-1.docx
COUNCIL THEORY 2018 ANSWERED-1.docx
 
15 soal basic science
15 soal basic science15 soal basic science
15 soal basic science
 
CC.pdf
CC.pdfCC.pdf
CC.pdf
 
Pneumonia cases
Pneumonia casesPneumonia cases
Pneumonia cases
 
Review course 2013 answer key.anand.tiwari
Review course 2013 answer key.anand.tiwariReview course 2013 answer key.anand.tiwari
Review course 2013 answer key.anand.tiwari
 
Clinical cases (2) june 9, 13
Clinical cases  (2) june 9, 13Clinical cases  (2) june 9, 13
Clinical cases (2) june 9, 13
 
IVMS Comprehensive -USMLE-type- Pharmacology-#1-2012 Examination
IVMS Comprehensive -USMLE-type- Pharmacology-#1-2012 ExaminationIVMS Comprehensive -USMLE-type- Pharmacology-#1-2012 Examination
IVMS Comprehensive -USMLE-type- Pharmacology-#1-2012 Examination
 

Recently uploaded

URLs and Routing in the Odoo 17 Website App
URLs and Routing in the Odoo 17 Website AppURLs and Routing in the Odoo 17 Website App
URLs and Routing in the Odoo 17 Website AppCeline George
 
Introduction to ArtificiaI Intelligence in Higher Education
Introduction to ArtificiaI Intelligence in Higher EducationIntroduction to ArtificiaI Intelligence in Higher Education
Introduction to ArtificiaI Intelligence in Higher Educationpboyjonauth
 
Crayon Activity Handout For the Crayon A
Crayon Activity Handout For the Crayon ACrayon Activity Handout For the Crayon A
Crayon Activity Handout For the Crayon AUnboundStockton
 
Hybridoma Technology ( Production , Purification , and Application )
Hybridoma Technology  ( Production , Purification , and Application  ) Hybridoma Technology  ( Production , Purification , and Application  )
Hybridoma Technology ( Production , Purification , and Application ) Sakshi Ghasle
 
Sanyam Choudhary Chemistry practical.pdf
Sanyam Choudhary Chemistry practical.pdfSanyam Choudhary Chemistry practical.pdf
Sanyam Choudhary Chemistry practical.pdfsanyamsingh5019
 
Measures of Central Tendency: Mean, Median and Mode
Measures of Central Tendency: Mean, Median and ModeMeasures of Central Tendency: Mean, Median and Mode
Measures of Central Tendency: Mean, Median and ModeThiyagu K
 
BASLIQ CURRENT LOOKBOOK LOOKBOOK(1) (1).pdf
BASLIQ CURRENT LOOKBOOK  LOOKBOOK(1) (1).pdfBASLIQ CURRENT LOOKBOOK  LOOKBOOK(1) (1).pdf
BASLIQ CURRENT LOOKBOOK LOOKBOOK(1) (1).pdfSoniaTolstoy
 
18-04-UA_REPORT_MEDIALITERAСY_INDEX-DM_23-1-final-eng.pdf
18-04-UA_REPORT_MEDIALITERAСY_INDEX-DM_23-1-final-eng.pdf18-04-UA_REPORT_MEDIALITERAСY_INDEX-DM_23-1-final-eng.pdf
18-04-UA_REPORT_MEDIALITERAСY_INDEX-DM_23-1-final-eng.pdfssuser54595a
 
Employee wellbeing at the workplace.pptx
Employee wellbeing at the workplace.pptxEmployee wellbeing at the workplace.pptx
Employee wellbeing at the workplace.pptxNirmalaLoungPoorunde1
 
Concept of Vouching. B.Com(Hons) /B.Compdf
Concept of Vouching. B.Com(Hons) /B.CompdfConcept of Vouching. B.Com(Hons) /B.Compdf
Concept of Vouching. B.Com(Hons) /B.CompdfUmakantAnnand
 
Introduction to AI in Higher Education_draft.pptx
Introduction to AI in Higher Education_draft.pptxIntroduction to AI in Higher Education_draft.pptx
Introduction to AI in Higher Education_draft.pptxpboyjonauth
 
PSYCHIATRIC History collection FORMAT.pptx
PSYCHIATRIC   History collection FORMAT.pptxPSYCHIATRIC   History collection FORMAT.pptx
PSYCHIATRIC History collection FORMAT.pptxPoojaSen20
 
Software Engineering Methodologies (overview)
Software Engineering Methodologies (overview)Software Engineering Methodologies (overview)
Software Engineering Methodologies (overview)eniolaolutunde
 
Micromeritics - Fundamental and Derived Properties of Powders
Micromeritics - Fundamental and Derived Properties of PowdersMicromeritics - Fundamental and Derived Properties of Powders
Micromeritics - Fundamental and Derived Properties of PowdersChitralekhaTherkar
 
MENTAL STATUS EXAMINATION format.docx
MENTAL     STATUS EXAMINATION format.docxMENTAL     STATUS EXAMINATION format.docx
MENTAL STATUS EXAMINATION format.docxPoojaSen20
 
“Oh GOSH! Reflecting on Hackteria's Collaborative Practices in a Global Do-It...
“Oh GOSH! Reflecting on Hackteria's Collaborative Practices in a Global Do-It...“Oh GOSH! Reflecting on Hackteria's Collaborative Practices in a Global Do-It...
“Oh GOSH! Reflecting on Hackteria's Collaborative Practices in a Global Do-It...Marc Dusseiller Dusjagr
 
POINT- BIOCHEMISTRY SEM 2 ENZYMES UNIT 5.pptx
POINT- BIOCHEMISTRY SEM 2 ENZYMES UNIT 5.pptxPOINT- BIOCHEMISTRY SEM 2 ENZYMES UNIT 5.pptx
POINT- BIOCHEMISTRY SEM 2 ENZYMES UNIT 5.pptxSayali Powar
 
Presentation by Andreas Schleicher Tackling the School Absenteeism Crisis 30 ...
Presentation by Andreas Schleicher Tackling the School Absenteeism Crisis 30 ...Presentation by Andreas Schleicher Tackling the School Absenteeism Crisis 30 ...
Presentation by Andreas Schleicher Tackling the School Absenteeism Crisis 30 ...EduSkills OECD
 
APM Welcome, APM North West Network Conference, Synergies Across Sectors
APM Welcome, APM North West Network Conference, Synergies Across SectorsAPM Welcome, APM North West Network Conference, Synergies Across Sectors
APM Welcome, APM North West Network Conference, Synergies Across SectorsAssociation for Project Management
 

Recently uploaded (20)

URLs and Routing in the Odoo 17 Website App
URLs and Routing in the Odoo 17 Website AppURLs and Routing in the Odoo 17 Website App
URLs and Routing in the Odoo 17 Website App
 
Introduction to ArtificiaI Intelligence in Higher Education
Introduction to ArtificiaI Intelligence in Higher EducationIntroduction to ArtificiaI Intelligence in Higher Education
Introduction to ArtificiaI Intelligence in Higher Education
 
Crayon Activity Handout For the Crayon A
Crayon Activity Handout For the Crayon ACrayon Activity Handout For the Crayon A
Crayon Activity Handout For the Crayon A
 
Hybridoma Technology ( Production , Purification , and Application )
Hybridoma Technology  ( Production , Purification , and Application  ) Hybridoma Technology  ( Production , Purification , and Application  )
Hybridoma Technology ( Production , Purification , and Application )
 
Sanyam Choudhary Chemistry practical.pdf
Sanyam Choudhary Chemistry practical.pdfSanyam Choudhary Chemistry practical.pdf
Sanyam Choudhary Chemistry practical.pdf
 
Staff of Color (SOC) Retention Efforts DDSD
Staff of Color (SOC) Retention Efforts DDSDStaff of Color (SOC) Retention Efforts DDSD
Staff of Color (SOC) Retention Efforts DDSD
 
Measures of Central Tendency: Mean, Median and Mode
Measures of Central Tendency: Mean, Median and ModeMeasures of Central Tendency: Mean, Median and Mode
Measures of Central Tendency: Mean, Median and Mode
 
BASLIQ CURRENT LOOKBOOK LOOKBOOK(1) (1).pdf
BASLIQ CURRENT LOOKBOOK  LOOKBOOK(1) (1).pdfBASLIQ CURRENT LOOKBOOK  LOOKBOOK(1) (1).pdf
BASLIQ CURRENT LOOKBOOK LOOKBOOK(1) (1).pdf
 
18-04-UA_REPORT_MEDIALITERAСY_INDEX-DM_23-1-final-eng.pdf
18-04-UA_REPORT_MEDIALITERAСY_INDEX-DM_23-1-final-eng.pdf18-04-UA_REPORT_MEDIALITERAСY_INDEX-DM_23-1-final-eng.pdf
18-04-UA_REPORT_MEDIALITERAСY_INDEX-DM_23-1-final-eng.pdf
 
Employee wellbeing at the workplace.pptx
Employee wellbeing at the workplace.pptxEmployee wellbeing at the workplace.pptx
Employee wellbeing at the workplace.pptx
 
Concept of Vouching. B.Com(Hons) /B.Compdf
Concept of Vouching. B.Com(Hons) /B.CompdfConcept of Vouching. B.Com(Hons) /B.Compdf
Concept of Vouching. B.Com(Hons) /B.Compdf
 
Introduction to AI in Higher Education_draft.pptx
Introduction to AI in Higher Education_draft.pptxIntroduction to AI in Higher Education_draft.pptx
Introduction to AI in Higher Education_draft.pptx
 
PSYCHIATRIC History collection FORMAT.pptx
PSYCHIATRIC   History collection FORMAT.pptxPSYCHIATRIC   History collection FORMAT.pptx
PSYCHIATRIC History collection FORMAT.pptx
 
Software Engineering Methodologies (overview)
Software Engineering Methodologies (overview)Software Engineering Methodologies (overview)
Software Engineering Methodologies (overview)
 
Micromeritics - Fundamental and Derived Properties of Powders
Micromeritics - Fundamental and Derived Properties of PowdersMicromeritics - Fundamental and Derived Properties of Powders
Micromeritics - Fundamental and Derived Properties of Powders
 
MENTAL STATUS EXAMINATION format.docx
MENTAL     STATUS EXAMINATION format.docxMENTAL     STATUS EXAMINATION format.docx
MENTAL STATUS EXAMINATION format.docx
 
“Oh GOSH! Reflecting on Hackteria's Collaborative Practices in a Global Do-It...
“Oh GOSH! Reflecting on Hackteria's Collaborative Practices in a Global Do-It...“Oh GOSH! Reflecting on Hackteria's Collaborative Practices in a Global Do-It...
“Oh GOSH! Reflecting on Hackteria's Collaborative Practices in a Global Do-It...
 
POINT- BIOCHEMISTRY SEM 2 ENZYMES UNIT 5.pptx
POINT- BIOCHEMISTRY SEM 2 ENZYMES UNIT 5.pptxPOINT- BIOCHEMISTRY SEM 2 ENZYMES UNIT 5.pptx
POINT- BIOCHEMISTRY SEM 2 ENZYMES UNIT 5.pptx
 
Presentation by Andreas Schleicher Tackling the School Absenteeism Crisis 30 ...
Presentation by Andreas Schleicher Tackling the School Absenteeism Crisis 30 ...Presentation by Andreas Schleicher Tackling the School Absenteeism Crisis 30 ...
Presentation by Andreas Schleicher Tackling the School Absenteeism Crisis 30 ...
 
APM Welcome, APM North West Network Conference, Synergies Across Sectors
APM Welcome, APM North West Network Conference, Synergies Across SectorsAPM Welcome, APM North West Network Conference, Synergies Across Sectors
APM Welcome, APM North West Network Conference, Synergies Across Sectors
 

NIMSET 2013 Questions

  • 1. ORIGINAL COPY 1 of 40 NIZAM’S INSTITUTE OF MEDICAL SCIENCES Punjagutta, Hyderabad – 500 082, A.P., India. Set: C NIMSET 2013 Question Booklet Entrance Examination for Admission to Postgraduate courses in MD/MS 24 February 2013 Marks 150 Time: 2 hours 30 min • Please verify whether the number printed on this question booklet corresponds to your hall-ticket number • Please read the instructions on the OMR-Answer sheet carefully • Each question in this booklet is followed by four alternative answers Select the one that is most appropriate and darken the corresponding Oval with HB pencil in the OMR-Answer sheet • Please return this question booklet along with your OMR-Answer sheet • Answer sheets not accompanied with the question booklets will not be corrected • Wrong answers carry negative marks Q1. The following drug does not act as treatment for acinetobacter infections
  • 2. ORIGINAL COPY 2 of 40 A) Sulbactam B) Ertapenem C) Doripenem D) Meropenem Q2. A 24 year old girl is brought to emergency, with history of consuming unknown substance. She complains of headache, giddiness and tinnitus. Her lips ,mouth and tongue are corroded, white and hardened. Urine output is scanty and black in color. The most probable substance consumed is A) Sulfuric acid B) Carbolic acid C) Hydrochloric acid D) Oxalic acid Q3. A child with an absent thumb and hypoplastic radius has also required treatment for a symptomatic arrhythmia and VSD. What is the most likely underlying unifying diagnosis? A) Down syndrome B) Chondroectodermal dysplasia (Ellis-van-Crevald syndrome) C) Holt-Oram syndrome D) Hailey-Hailey syndrome Q4. Labyrinthine artery mostly arises from A) Posterior inferior cerebellar artery (PICA) B) Anterior inferior cerebellar artery (AICA) C) Basilar artery D) Vertebral Artery Q5. Hebra nose is seen in A) Leprosy B) Rhinosporidiosis C) Rhinitis medicamentosa D) Rhinoscleroma Q6. Complication/complications of ulcerative blepharitis: 1. Madarosis
  • 3. ORIGINAL COPY 3 of 40 2. Fibrosis 3. Trichiasis 4. Optic neuritis A) Only 1,2 and 3 are correct B) Only 1 and 3 are correct C) Only 2 and 4 are correct D) Only 4 is correct Q7. A 26 year old lady came with the complaints of fever with altered sensorium and reduced urine output of 1 week duration. On evaluation her hemoglobin was 7 gm/dl, platelet count of 30,000/cu.mm with peripheral smear showing schistocytes. Serum creatinine was 3mg/dl. Prothrombin time and activated partial thromboplastin time were normal. Enzyme most likely to be deficient in this situation is A) Hexokinase B) Pyruvate kinase C) ADAMTS 13 D) Glucose-6 phosphate dehydrogenase Q8. A ten year old girl presented to the emergency medicine department with vague upper abdominal pain and jaundice. On examination she was febrile and a tender mass was felt in the right upper quadrant of the abdomen without signs of peritonitis. Laboratory studies revealed raised serum bilirubin and serum alkaline phosphatase. The most probable diagnosis is A) Perforated duodenal ulcer B) Meckels diverticulum with sepsis C) Cholelithiasis and choledocholithiasis D) Choledochal cyst Q9. Transmission of Clostridium difficile in clinical practice can be prevented by 1. Use of alcohol hand gels 2. Use of gloves by health care workers 3. Increasing the use of 2nd and 3rd generation cephalosporins 4. Use of hypochlorite solution for environmental decontamination A) Only 1, 2 and 3 are correct B) Only 1 and 3 are correct C) Only 2 and 4 are correct D) Only 4 is correct Q10. Regarding 'Wernicke's encephalopathy' Assertion: In acute presentation, patient has nystagmus, confusion, ophthalmoplegia and ataxia Reason: Caused by deficiency of thiamine
  • 4. ORIGINAL COPY 4 of 40 A) Both Assertion and Reason are true and Reason is the correct explanation for Assertion B) Both Assertion and Reason are true and Reason is not the correct explanation for Assertion C) Assertion is true, but Reason is false D) Assertion is false, but Reason is true Q11. Regarding amiodarone Assertion: it is indicated for therapy in patients with recurrent ventricular tachycardia or fibrillation. Reason: decreases refractoriness of all cardiac tissues. A) Both Assertion and Reason are true, and Reason is the correct explanation for Assertion B) Both Assertion and Reason are true, and Reason is not the correct explanation for Assertion C) Assertion is true, but Reason is false D) Assertion is false, but Reason is true Q12. Antiretroviral therapy should be initiated 1) In all those who are HIV infected with CD4 count ≤ 350 cells/cu.mm 2) In HIV infected individuals with concurrent Hepatitis B virus infection irrespective of CD4 count 3) With at least three drugs simultaneously for entire duration of treatment. 4) In all HIV infected adults and children irrespective of CD4 count. A) 1, 2 & 3 are correct B) Only 1 & 3 are correct C) Only 2 & 4 are correct D) Only 4 is correct Q13. Regarding the management of myxedema coma Assertion: parenteral hydrocortisone should be administered Reason: there is concomittant autoimmune failure of adrenal glands in all patients with myxedema coma
  • 5. ORIGINAL COPY 5 of 40 A) Both Assertion and Reason are true and Reason is the correct explanation for the Assertion B) Both Assertion and Reason are true and Reason is not the correct explanation for the Assertion C) Assertion is true but Reason is false D) Assertion is false but Reason is true Q14. Cisplatin, an anticancer chemotherapeutic agent, is a A) Alkylating agent B) Antimetabolite C) Antitumor antibiotic D) Tyrosine kinase inhibitor Q15. Which of these is not an inhibitor of fibrinolysis A) Plasminogen Activator Inhibitor B) Tissue Plasminogen Activator C) α 2 - Antiplasmin D) Thrombin Activatable Fibrinolysis Inhibitor Q16. A patient with known ulcerative colitis was admitted as an emergency. The plain abdominal radiograph showed dilatation and mucosal islands in keeping with toxic megacolon. In which part of the large bowel are these changes most commonly seen? A) Caecum B) Ascending colon C) Transverse colon D) Descending colon Q17. Regarding management of prosthetic joint infections A) Prosthesis need not be replaced in most cases to clear the infection B) Treatment includes surgery and high doses of parenteral antibiotics for up to 6 weeks C) Implantation of the new prosthesis can be done immediately D) Addition of rifampicin is not useful in patients with staphylococcal prosthetic joint infection Q18. Regarding treatment of yeast and dermatophyte fungal infection of skin Assertion: Ketoconazole is an effective drug but reserved for severe infections Reason: It can cause hepatotoxicity and thrombocytopenia.
  • 6. ORIGINAL COPY 6 of 40 A) Both Assertion and Reason are true and Reason is correct explanation of Assertion. B) Both Assertion and Reason are true and Reason is not correct explanation of Assertion. C) Assertion is true and Reason is false D) Assertion is false and Reason is true Q19. Which of the following is not true regarding a case-control study A) It is useful in studying rare diseases B) It can measure incidence rates C) It is useful in studying multiple risk factors D) It is a retrospective study Q20. In the graph shown below depicting the effect of increasing minimum alveolar concentration (MAC)of an inhalational anesthetic agent on different components of anesthesia, the components 1,2, and 3 correspond to which one of the following respectively A) Unconsciousness, immobility, amnesia B) Unconsciousness, amnesia, immobility, C) Immobility, unconsciousness, amnesia, D) Amnesia, unconsciousness, immobility Q21. Which of the following correctly describes neurocysticercosis A) Infection by Taenia solium and epilepsy B) Infection by Echinoccocus and cyst formation C) Always associated with immunodeficient state
  • 7. ORIGINAL COPY 7 of 40 D) Formation of pyogenic brain abscess Q22. A part of an mRNA is shown below. A mutation of the template DNA results in a change in codon 66 to GGA. The type of mutation is 63 64 65 66 67 GAC CAG UAG GGC UAA A) Missense B) Silent C) Nonsense D) Frame shift Q23. True statement/statements regarding MRSA 1. Mechanism of resistance is by production of beta lactamase. 2. It is susceptible to 3rd and greater generation cephalosporins. 3. Beta-lactam and beta lactamase inhibitor combinations are effective against MRSA. 4. It is resistant to cloxacillin. A) Only 1, 2 and 3 are correct B) Only 1 and 3 are correct C) Only 2 and 4 are correct D) Only 4 is correct Q24. Drugs that carry risk of clinical hemolysis in persons with G6PD deficiency include 1. Carbamazepine 2. Acetaminophen 3. Phenytoin 4. Nalidixic acid A) Only 1, 2 and 3 are correct B) Only 1 and 3 are correct C) Only 2 and 4 are correct D) Only 4 is correct Q25. Regarding seminoma of the testes, the following statement is correct A) Peak incidence of seminoma is in the second decade of life B) Active lymphocytic infiltration of tumour suggests good host response and better outcome.
  • 8. ORIGINAL COPY 8 of 40 C) Early suspicion of the disease can be made out by inguinal lymphadenitis. D) Surgery is the treatment of choice. Q26. Low molecular weight (LMW) heparins have replaced continuous heparin (UFH) infusions in management of thromboembolic conditions as 1) aPTT monitoring is not required for LMW heparins. 2) LMW Heparins can be given in patients with severe renal failure. 3) LMW Heparins have long biological half-lives 4) Heparin induced thrombocytopenia does not occur with LMW Heparins A) 1, 2 & 3 are correct B) Only 1 & 3 are correct C) Only 2 & 4 are correct D) Only 4 is correct Q27. The complication of Uterine myoma that develops most frequently during pregnancy is, A) Calcareous degeneration B) Hyaline degeneration C) Red degeneration D) Cystic degeneration Q28. A 68-year old male presented with easy fatigability, weight loss and anorexia. He was found to have generalized lymphadenopathy and mild hepatosplenomegaly. His Hemoglobin was 12.0gm/dl, TLC- 130,000/cmm and platelets were normal. The blood smear is depicted in the picture below
  • 9. ORIGINAL COPY 9 of 40 What is the likely diagnosis? A) Hairy cell leukemia B) Chronic Myeloid leukemia C) Chronic lymphocytic leukemia D) Idiopathic myelofibrosis Q29. The condition in which muscles are contracted during life, become stiff and rigid immediately after death, without passing into the stage of primary relaxation is A) Cadaveric Rigidity B) Heat Stiffening C) Cold Stiffening D) Cadaveric Spasm Q30. Regarding management of diabetic ketoacidosis Assertion: after the initial bolus of normal saline and hemodynamic stability are achieved intravenous fluids should be switched to 0.45% saline Reason: 0.45% saline reduces hypernatremia A) Both Assertion and Reason are true, and Reason is the correct explanation for Assertion B) Both Assertion and Reason are true, and Reason is not the correct explanation for Assertion C) Assertion is true, but Reason is false D) Assertion is false, but Reason is true Q31. The pain experienced by the woman who shed endometrium as a cast at the time of menstruation is called A) Spasmodic dysmenorrhea
  • 10. ORIGINAL COPY 10 of 40 B) Congestive dysmenorrhea C) Membranous dysmenorrhea D) Unilateral dysmenorrhea Q32. Regarding Ricocheting of a bullet, which of following statement is false A) Occurs with inferior firearms and low velocity bullets B) Produces large oval, triangular or cruciate entrance wound C) Burning, blackening and tattooing are seen around the entrance wound D) Abrasion collar is absent Q33. A new serum marker that increases in a particular disease state is being evaluated. The graph below depicts the relationship between sensitivity and false positive rate for different cut- off values for the marker concentration in the serum. Assuming that both correct diagnosis and ruling out the disease are equally important clinically, which one of the values represents the appropriate cut-off value A) 200 B) 5 C) 75 D) 50 Q34. Regarding Bupivacaine, a local anaesthetic Assertion: It is cardiotoxic. Reason: It blocks cardiac sodium channels and produces ventricular arrhythmias
  • 11. ORIGINAL COPY 11 of 40 and myocardial depression. A) Assertion and Reason are true and Reason is the correct explanation of assertion. B) Assertion and Reason are true but Reason is not correct explanation of assertion C) Assertion is true and reason is False. D) Assertion is False and Reason is true. Q35. The most common cause of acute pancreatitis is A) Gall stones B) Alcoholism C) Malnutrition D) Abdominal trauma Q36. A low bioavailability of a drug may result from, 1. Failure of dosage form to disintegrate or dissolve in GI fluid 2. High first-pass metabolism 3. High active efflux transport of drug back into lumen. 4. High mucosal permeability. A) Only 1, 2 and 3 are correct B) Only 1 and 3 are correct C) Only 2 and 4 are correct D) Only 4 is correct. Q37. Sarcoidosis associated hypercalcemia is treated with A) Steroids B) Thiazide diuretics C) Bisphosphonates D) Alkalinization of urine Q38. The endolymph of inner ear is rich in A) Potassium ions B) Sodium ions C) Chloride ions D) Magnesium Q39. Cessation of smoking for as short time as 12-48 hrs A) Decreases carboxyhaemoglobin levels and nicotine levels B) Decreases sputum production
  • 12. ORIGINAL COPY 12 of 40 C) Improves pulmonary function tests D) Normalizes immune function and metabolism Q40. Which of the following is not a manifestation of Wilson disease? A) Mitral valve stenosis B) Hemolytic anemia C) Cirrhosis D) Dysarthria and dystonia Q41. Regarding the process of callus formation following a fracture A) Forms on periosteal surface only B) Forms in response to some movement at fracture site C) Lamellar bone is transformed into woven bone D) Most often results in cartilage formation Q42. A patient with bronchial asthma on bronchodilator therapy is scheduled for laparotomy. The most suitable pulmonary function test to assess improvement in lung function is A) Forced vital capacity B) Total lung capacity C) Peak expiratory flow rate D) Maximum breathing capacity Q43. Percentage of carbon-dioxide transported in carbaminohemoglobin form is A) 7% B) 23% C) 70% D) 35% Q44. A patient with which of the following attributes is the most preferred patient for bariatric surgery A) Nonambulatory patient with BMI >40 B) BMI between 30-35 with no comorbidities C) BMI>40 with no comorbidity D) Depression due to morbid obesity Q45. A 3 year male child presented with complaints of recurrent upper respiratory tract infections, recurrent skin boils from the age of 6months. He was a product of a consanguineous marriage. On examination he was malnourished, had nystagmus, hypopigmented hair, multiple skin boils, generalized lymphadenopathy and mild hepatosplenomegaly. His elder sibling also had similar complaints and died at the age of 5yrs. A blood examination done showed Hb of
  • 13. ORIGINAL COPY 13 of 40 9gm%, TLC 15,000, Platelet 1.5 lakhs , normochromic, normocytic picture, large inclusions in all the nucleated blood cells. The most probable diagnosis is A) Chediak -Higashi syndrome B) Chronic granulomatous disease C) Hyper Ig E syndrome D) Leukocyte adhesion deficiency Q46. Footling presentation is a form of A) Complete breech presentation B) Frank breech presentation C) Incomplete breech presentation D) Compound presentation Q47. In a child 2 months old with pertusis, Assertion: The disease is only diagnosed after classical whoop develops Reason: The catarrhal stage may last only few days A) Both Assertion and Reason are true, and Reason is the correct explanation for Assertion B) Both Assertion and Reason are true, and Reason is not the correct explanation for Assertion C) Assertion is true, but Reason is false D) Assertion is false, but Reason is true Q48. Regarding relationship between ovarian and breast cancer, Assertion: Women with ovarian cancer are at a higher risk for developing breast cancer and vice-versa Reason: Both malignancies have common oncogenic factors A) Both Assertion and Reason are true, and Reason is the correct explanation for Assertion B) Both Assertion and Reason are true, and Reason is not the correct explanation for Assertion C) Assertion is true, but Reason is false D) Assertion is false, but Reason is true Q49. In a normal child, head circumference at the age of 2 years is approximately A) 45 cms B) 48 cms C) 52 cms D) 54 cms
  • 14. ORIGINAL COPY 14 of 40 Q50. Following feature/features of indirect inguinal hernia differentiate it from direct inguinal hernia 1. Neck of indirect hernia is lateral to epigastric vessels 2. Indirect hernia occurs in the elderly 3. Surgery of choice for indirect hernia in young adults is herniotomy alone 4. Indirect hernia never strangulates A) Only 1, 2 and 3 are correct B) Only 1 and 3 are correct C) Only 2 and 4 are correct D) Only 4 is correct Q51. Systemic inflammatory response syndrome (SIRS) is the body's systemic response to severe infection. Incorrect statement regarding SIRS is A) Lipopolysaccharide endotoxin from the walls of dying gram negative bacilli is responsible B) Interleukin-1 and tumour necrosis factor are mediators of inflammation C) Leukopenia D) Activated neutrophils adhere to vascular endothelium and decrease vascular permeability Q52. The rate constant of a biochemical reaction follows A) Exponential increase with increase in temperature B) Exponential decrease with increase in temperature C) Linear increase with increase in temperature D) Linear decrease with increase in temperature Q53. An acute extradural hematoma is seen on CT as A) Hyperdense concave lesion B) Isodense lentiform (biconvex) lesion C) Hyperdense lentiform (biconvex) lesion D) Hypodense lentiform (biconvex) lesion Q54. Regarding enteric fever 1. Complications such as gastrointestinal bleeding and intestinal perforation occur most commonly in the 2nd week of illness. 2. Rash (rose spots) in enteric fever occurs late in the course of the disease. 3. The definitive diagnosis of enteric fever rests on the demonstration of febrile
  • 15. ORIGINAL COPY 15 of 40 agglutinins by Widal test 4. High dose fluoroquinolone therapy for 7 days for Nalidixic acid resistant (NAR) enteric fever has been associated with delayed resolution of fever and high rates of fecal carriage during convalescence. A) Only 1, 2 and 3 are correct. B) Only 1 and 3 are correct C) Only 2 and 4 are correct D) Only 4 is correct Q55. The mutation due to substitution of a pyrimidine by a purine or vice versa is known as A) Transition B) Transversion C) Replacement D) Dynamic mutation Q56. Regarding treatment of acute adrenal insufficiency Assertion: glucocorticoids such as prednisolone are preferred Reason: long acting glucocorticoids such as prednisolone result in increased glucocorticoid exposure due to extended glucocorticoid receptor activation A) Both Assertion and Reason are true, and Reason is the correct explanation for Assertion B) Both Assertion and Reason are true, and Reason is not the correct explanation for Assertion C) Assertion is true, but Reason is false D) Assertion is false, but Reason is true Q57. Regarding Propofol which one is not correct A) Has a pH of 7.0 B) Insoluble in water C) Larger dosage is required in pediatric population D) Propofol increases intracranial pressure Q58. A 25-year old man, hailing from Eastern Uttar Pradesh, presented with fever, weight loss, hepatosplenomegaly and pancytopenia. Bone marrow examination revealed the pathology as depicted in the figure.
  • 16. ORIGINAL COPY 16 of 40 A) Plasmodium falciparum B) Candida C) Leishmania donovani D) Mycobacterium tuberculosis Q59. Clinical features of Argyll Robertson pupil: A) Light reflex is present B) Seen in diabetes C) Reaction to convergence and accommodation present D) Pupil is dilated Q60. During early pregnancy the hormone that mimics the action of LH is A) Estrogen B) Progesterone C) HCG D) Prolactin Q61. In granulomatous inflammation due to tuberculosis, which of these statements is correct? 1 The granuloma consists of aggregates of epithelioid cells, lymphocytes and fibroblasts 2. Epithelioid cells fuse to form Langhans-type giant cells 3. There is central caseous necrosis 4. Numerous eosinophils are seen A) Only 1, 2 & 3 are correct B) Only 1 and 3 are correct C) Only 2 and 4 are correct D) Only 4 is correct Q62. In spinal cord injury, bladder drainage technique with the lowest infection rate is A) Clean intermittent catheterization B) Supra-pubic drainage C) Indwelling catheter D) Condom catheter
  • 17. ORIGINAL COPY 17 of 40 Q63. A 17 year old boy presented with crampy abdominal pain and a history of darkened stools over the last few weeks. Physical examination was normal other than some vague central abdominal tenderness and pigmentation of lower lip. An urgent ultrasound showed a probable intussusception. Following reduction a follow up small bowel study showed multiple broad based polyps mainly in the jejunum and ileum. What is the most likely diagnosis? A) Familial adenomatous polyposis B) Cowden syndrome C) Juvenile polyposis D) Peutz-Jeghers syndrome Q64. Computed tomographic scan of the upper abdomen in a young girl who is complaining of persistent upper abdominal pain has shown the following image Which of the following statement holds true A) Non vegetarian diet is responsible for this B) Ultrasound guided needle aspiration cures the disease C) Needs intervention only if it ruptures into the peritoneal cavity and causes peritonitis D) Laparotomy and pericystectomy are needed. Q65. Brown-Sequard Syndrome consists of A) Motor paralysis and loss of dorsal column sensations on the same side below the lesion and loss of pain, temperature and crude touch on the opposite side. B) Motor paralysis on the opposite side of the lesion. C) Same as in A with loss of dorsal column sensations on the opposite side below the lesion.
  • 18. ORIGINAL COPY 18 of 40 D) Same as in A with no Motor Paralysis. Q66. Cobra venom is mainly A) Hepatotoxic B) Neurotoxic C) Ototoxic D) Haematotoxic Q67. The parotid salivary gland receives its parasympathetic innervation from which one of the following nerves A) Vagus B) Facial C) Glossopharyngeal D) Oculomotor Q68. Regarding tubercular arthritis A) Primarily involves small joints of the hand B) Systemic symptoms are seen in all patients C) Usually associated with active pulmonary tuberculosis D) Predominant cell type in synovial fluid is neutrophil Q69. Regarding use of phototherapy in the management of neonatal hyperbilirubinemia: 1. Light in the blue range is most effective in reducing bilirubin levels 2. Ultraviolet radiation is preferred for phototherapy 3. Dark skin does not reduce the efficacy of phototherapy 4. Phototherapy helps convert unconjugated to conjugated bilirubin A) Only 1, 2 and 3 are correct B) Only 1 and 3 are correct C) Only 2 and 4 are correct D) All (1, 2, 3 and 4) are correct Q70. Regarding Sydenham's chorea which is not true A) ASO titres can be normal at the time of manifestation of chorea B) Psychiatric disturbances can occur in the long term following chorea C) Neurological deficits persist for upto 10 years D) Reported incidence 5-35% in rheumatic fever
  • 19. ORIGINAL COPY 19 of 40 Q71. Regarding p value Assertion: A p value of 0.05 is more significant than 0.01 Reason: It gives an estimate of the probability of a result occurring by chance. A) Both Assertion and Reason are true and Reason is the correct explanation for Assertion B) Both Assertion and Reason are true and Reason is not the correct explanation for Assertion C) Assertion is true but Reason is false D) Reason is true but Assertion is false Q72. 'My head is missing'; 'I have no body'; 'I am dead'- these are symptoms described in A) Hypochondriasis B) Persecutory Delusion C) Grandiose Delusion D) Nihilistic Delusion Q73. Diagnostic biochemical investigation in Paget's disease is by estimating A) 24 hour urinary calcium excretion B) 24 hour urinary Phosphorous excretion C) Urinary alkaline phosphatase D) 24 hour urinary hydroxy proline Q74. Korner's Septum is present in A) Petrosquamous suture B) Tympanomastoid suture C) Tympano Squamous suture D) Tympano petrous petrous suture Q75. Pre ganglionic para sympathetic fibres to otic ganglion are carried by: 1. Auriculo-temporal nerve. 2. Temporal branch of facial nerve. 3. Auricular branch of vagus nerve. 4. Lesser petrosal nerve.
  • 20. ORIGINAL COPY 20 of 40 A) Only 1, 2 & 3 are correct. B) Only 1 and 3 are correct. C) Only 2 & 4 are correct. D) Only 4 is correct. Q76. Regarding proton pump inhibitors: Assertion: Proton pump inhibitors should ideally be given 30 minutes before meals. Reason: Acidic pH in gastric lumen increases absorption of proton pump inhibitors. A) Both Assertion and Reason are true, and Reason is the correct explanation for Assertion B) Both Assertion and Reason are true, and Reason is not the correct explanation for Assertion C) Assertion is true, but Reason is false D) Assertion is false, but Reason is true Q77. In severe diarrhea, dehydration and electrolyte imbalance pose principal risk especially in children. Assertion: Oral Rehydration therapy is cornerstone for patients with acute illnesses resulting in significant diarrhea. Reason: Nutrient linked cotransport of water and electrolytes remains intact in small bowel in most cases of acute diarrhea. A) Both Assertion and Reason are true, and Reason is the correct explanation for Assertion B) Both Assertion and Reason are true, and Reason is not the correct explanation for Assertion C) Assertion is true, but Reason is false D) Assertion is false, but Reason is true Q78. A 39 year old was imaged following sudden onset right hemiplegia. A CT showed subtle atrophy of the left occipital lobe and unusual gyral calcification. MRI demonstrated prominent pial and deep medullary veins in the region with a prominent left choroid plexus. What is the underlying aetiology? A) Tuberous sclerosis
  • 21. ORIGINAL COPY 21 of 40 B) Ataxia telangiectasia C) Von Hippel-Lindau syndrome D) Sturge-Weber syndrome Q79. A 6 week old infant is admitted to the hospital with jaundice. On investigating for jaundice, he has total serum bilirubin of 13 mg/dl and direct fraction of 2 mg/dl. Condition/conditions which can be suspected 1. Choledochal cyst 2. Gilbert Syndrome 3. Galactosemia 4. Crigler - Najjar Syndrome A) Only 1, 2 and 3 are correct B) Only 1 and 3 are correct C) Only 2 and 4 are correct D) Only 4 is correct Q80. Regarding congenital defect in the development of face, Assertion: Oblique facial cleft extends from the medial angle of eye to mouth (upper lip) Reason: Maxillary process does not fuse with the palatine process A) Both Assertion and Reason are true, and Reason is the correct explanation for Assertion B) Both Assertion and Reason are true, and Reason is not the correct explanation for Assertion C) Assertion is true, but Reason is false D) Assertion is false, but Reason is true Q81. Regarding anticholinergics in bronchial asthma Assertion: They are equally effective as Beta2 agonists in asthma therapy Reason: They act by inhibiting only the cholinergic reflex component of bronchoconstriction.
  • 22. ORIGINAL COPY 22 of 40 A) Both Assertion and Reason are true, and Reason is the correct explanation for the Assertion B) Both Assertion and Reason are true, and Reason is not the correct Explanation for Assertion C) Assertion is true, but Reason is false D) Assertion is false, but Reason is true Q82. In humans if both noradrenergic and cholinergic systems are blocked, the approximate heart rate would be approximately A) 70 B) 50 C) 100 D) 120 Q83. Left atrial enlargement can be recognized on a chest radiograph as 1. Rounding of left heart border 2. Double contouring of right cardiac shadow 3. Elevated cardiac apex 4. Widening of carina A) Only 1, 2 and 3 are correct B) Only 1 and 3 are correct C) Only 2 and 4 are correct D) Only 4 is correct Q84. Among the Duke criteria for the clinical diagnosis of infective endocarditis, the major criteria include 1. Growth of a typical microorganism for Infective endocarditis from two separate blood cultures. 2. Osler's nodes
  • 23. ORIGINAL COPY 23 of 40 3. Single positive blood culture for Coxiella burnetii. 4. Fever ≥ 38.00 C(≥ 100.40 F) A) Only 1, 2 and 3 are correct. B) Only 1 and 3 are correct C) Only 2 and 4 are correct D) Only 4 is correct Q85. Which of the following is not a mechanism of action of Mifepristone (RU-486) A) Competes with progesterone receptors B) Causes sloughing of endometrium C) Causes uterine contraction and softening of cervix D) Has Glucocorticoid and Androgenic action. Q86. Regarding use of tamoxifen in patients of ER positive metastatic breast cancer Assertion: Patients with CYP2D6 polymorphisms that reduce its activity, are associated with higher risk of disease relapse. Reason: Tamoxifen is metabolized in liver by CYP2D6 into inactive metabolites, 4-OH Tamoxifen and endoxifen A) Both Assertion and Reason are true, and Reason is the correct explanation for Assertion B) Both Assertion and Reason are true, and Reason is not the correct explanation for Assertion C) Assertion is true, but Reason is false D) Assertion is false, but Reason is true Q87. Which of the following is not a risk factor for Candidial Vaginitis A) Prolonged steroid therapy B) Type II diabetes mellitus C) Thin statured woman D) Long term antibiotic therapy Q88. Identify the abnormality in the following chest radiograph
  • 24. ORIGINAL COPY 24 of 40 A) Fracture of left clavicle B) Fracture of 3rd rib Right side C) Bilateral cervical ribs D) Anterior Mediastinal Mass Q89. Which of the following is a component of the massive transfusion resuscitation protocol used to improve survival in the setting of severe trauma A) Use of large volume of crystalloids B) Use of whole blood for immediate resuscitation C) Use of unmatched O negative blood for immediate resuscitation D) For patients requiring more than 2 units of packed red blood cells 2 units of FFP should be given Q90. A 19 year old girl comes to out-patient with Primary amenorrhea. On examination, she has well developed Secondary sexual characters. FSH, LH levels are found to be normal and karyotyping is 46 XX. Ultrasound shows absent uterus and vagina. The diagnosis is A) Turner syndrome B) Savage syndrome. C) Rokitansky-Kustner Hauser syndrome. D) Kallmans syndrome. Q91. Regarding advice of following a traditional low fat diet Assertion: It is a primordial preventive measure for coronary artery disease
  • 25. ORIGINAL COPY 25 of 40 Reason: It reduces the risk of conversion of angina to myocardial infarction A) Both Assertion and Reason are true, and Reason is the correct explanation for Assertion B) Both Assertion and Reason are true, and Reason is not the correct explanation for Assertion C) Assertion is true, but Reason is false D) Assertion is false, but Reason is true Q92. A 50 year old man with history of persistent shoulder pain had a jerking injury of the shoulder. On examination, 6 weeks later, he is unable to abduct his arm. Passive abduction was normal and he could keep the arm up once it had been raised above the right angle. This suggests A) Partial tear of rotator cuff B) Full thickness tear of rotator cuff C) Isolated infraspinatus weakness D) Isolated subscapularis weakness Q93. Second degree inversion of uterus is A) Uterine fundus just inverts into the uterine cavity B) Uterine fundus is at the cervix C) Uterine fundus protrudes through cervix and lies in vagina D) Whole of uterus is inverted and protrudes through the introitus Q94. Which of the following anthropometric parameters is not used as an indicator of the nutritional status of a toddler? A) Mid-upper arm circumference B) Weight for height C) Occipito-frontal head circumference D) Weight for age Q95. Regarding breast self examination Assertion: Monthly breast self examination is recommended by American Cancer Society
  • 26. ORIGINAL COPY 26 of 40 Reason: Breast self examination is as sensitive as mammography A) Both Assertion and Reason are true, and reason is the correct explanation for assertion B) Both Assertion and reason are true, and Reason is not the correct explanation for assertion C) Assertion is true, but Reason is false D) Assertion is false, but Reason is true Q96. What is true in the given below data of weights of 9 school children chosen randomly. 20, 31, 31, 31, 25, 28, 35, 38, 31. A) Mean is 31 B) Range is 20-38 C) Median is 15 D) Mode is 15 Q97. In a fire-arm injury ' shored exit' is the one, where A) Exit wound is slightly smaller than the entry wound. B) Exit wound is larger than the entry wound. C) Both Exit and entry wounds edges are inverted D) Both entry and exit wounds have abrasion collar at the margins. Q98. The mechanism of whip-lash injury to cervical spine is essentially A) Sudden extension of the neck followed by flexion. B) Sudden flexion followed by extension C) Sudden distraction D) Axial loading Q99. Juxta glomerular apparatus consists of
  • 27. ORIGINAL COPY 27 of 40 A) Only 1, 2 & 3 are correct B) Only 1 & 3 are correct C) Only 2 & 4 are correct D) Only 4 correct Q100. Which of the following interstitial lung diseases is not associated with smoking? A) Desquamative interstitial pneumonitis B) Bronchiolitis obliterans organizing pneumonia C) Pulmonary Langerhans cell granulomatosis D) Respiratory bronchiolitis-interstitial lung disease Q101. Unilateral dysmenorrhea occurs in A) Fibroid uterus B) Rudimentary horn of bicornuate uterus C) Endometriotic Cyst D) Pelvic inflammatory disease, Q102. The cross-section of the original Kuntscher nail (K- Nail) designed for the femur is A) Triangular B) Circular C) Oval D) Clover leaf Q103. The following are indications for strongly considering cardiac surgical
  • 28. ORIGINAL COPY 28 of 40 intervention in patients with endocarditis. 1. Perivalvular extension of infection 2. Poorly responsive or relapsed endocarditis due to highly antibiotic-resistant enterococci or gram negative bacilli 3. Persistent unexplained fever(≥ 10days) in culture negative native valve endocarditis. 4. Vegetation of 5 mm in diameter. A) Only 1, 2 and 3 are correct. B) Only 1 and 3 are correct C) Only 2 and 4 are correct D) Only 4 is correct Q104. A young patient presents with a massive injury to proximal duodenum, head of pancreas and distal common bile dyct. He is hemodyanamically stable. The surgical procedure of choice in this patient should be A) Roux - en-Y anastomosis B) Pancreaticoduodenectomy (Whipple's operation) C) Lateral tube jejunostomy D) Retrograde jejunostomy Q105. True statement regarding Frey's syndrome is A) Rare complication after parotidectomy B) Results from damage to autonomic innervation of the salivary gland with inappropriate regeneration of sympathetic nerve fibres that stimulate the sweat glands of the overlying skin C) Revision surgery and excision of affected skin and placement of skin graft is definite treatment D) Established Frey's syndrome can best be addressed by injection of botulinum toxin into the affected skin Q106. Identify the tissue given below
  • 29. ORIGINAL COPY 29 of 40 A) Dermis B) Epidermis C) Hair follicle D) Sebaceous gland Q107. Methylated purines and pyrimidines are characteristically present in A) mRNA B) hnRNA C) tRNA D) rRNA Q108. Mean corpuscular hemoglobin concentration (MCHC) is increased in following condition A) Hereditary spherocytosis B) Iron deficiency anemia C) Thalassemia D) Sickle cell diseases Q109. Which of the following is not a part of the Rotator Cuff? A) Supra Spinatus B) Teres Major C) Infra Spinatus D) Sub Scapularis Q110. Aminoglycoside antibiotics are
  • 30. ORIGINAL COPY 30 of 40 A) Bacteriostatic, effective against aerobic gram positive bacilli B) Bacteriostatic, effective against aerobic gram negative bacilli C) Bactericidal, effective against aerobic gram positive bacilli D) Bactericidal, effective against aerobic gram negative bacilli Q111. Regarding the laboratory diagnosis of M. tuberculosis infection 1. For patients with suspected pulmonary Tuberculosis, it is recommended that two or three sputum specimens collected early in the morning should be submitted for AFB smear and Mycobacterial culture. 2. Serologic tests based on detection of antibodies to a variety of mycobacterial antigens are not useful for the diagnosis of tuberculosis. 3. Tuberculin skin testing is of limited value in the diagnosis of active TB. 4. Fluorescence microscopy of specimens stained with auramine -rhodamine has the same sensitivity as light microscopy of specimens stained with Ziehl-Neelsen stain. A) Only 1, 2 and 3 are correct. B) Only 1 and 3 are correct C) Only 2 and 4 are correct D) Only 4 is correct Q112. An infant was vaccinated with measles vaccine at the age of 6 months due a measles outbreak in the village. Assertion: A second dose of measles vaccine is recommended as soon as possible after 9 months of age Reason: A different strain of virus can cause measles after an outbreak. A) Both Assertion and Reason are true and Reason is the correct explanation for Assertion B) Both Assertion and Reason are true and Reason is not the correct explanation for Assertion C) Assertion is true but Reason is false D) Reason is true but Assertion is false Q113. What chemical is used in identifying fungal infection of skin A) 20% potassium hydroxide B) Cyano acrylate C) Para amino benzoic acid D) 20% sulphuric acid Q114. Given ECG leads suggest
  • 31. ORIGINAL COPY 31 of 40 A) Left atrial enlargement B) Right ventricular hypertrophy C) Left ventricular enlargement D) Both right and left atrial enlargement Q115. Index/indices used in calculating human development index 1) Life expectancy at birth 2) Adult literacy rate 3) Gross domestic product (GDP) per capita 4) Maternal mortality rate A) Only 1, 2 and 3 are correct B) Only 1 and 3 are correct C) Only 2 and 4 are correct D) Only 4 is correct Q116. The diplopia chart in patient complaining of diplopia is given below Patients R Centre Patients L ↑ ↑ ↑↑ ↑ ↑ ↑ ↑↑ ↑ ↑ ↑ ↑↑ ↑ The most likely cause is A) Right lateral rectus palsy B) Right medial rectus palsy C) Left trochlear nerve lesion D) Right occulomotor nerve lesion Q117. A 25 years old lady comes to the hospital with history of 2 second trimester abortions. Both times she describes the events as being painless with expulsion of liquor first followed by
  • 32. ORIGINAL COPY 32 of 40 the immature fetus. The most probable cause of fetal loss is A) Congenital abnormalities B) Intrauterine growth Retardation (IUGR) C) Infection D) Cervical incompetence Q118. The only abductor of the vocal cord is A) Cricothyroid B) Vocalis C) Transverse arytenoid D) Posterior cricoarytenoid Q119. Layers of retina include/includes 1) Nerve fibre layer 2) Internal limiting membrane 3) Retinal pigment epithelium 4) Bowman's membrane A) Only 1, 2 and 3 are correct B) Only 1 and 3 are correct C) Only 2 and 4 are correct D) Only 4 is correct Q120. Submucosal glands are absent in A) Duodenum B) Oesophagus C) Trachea D) Appendix Q121. Which of the following is a structural analog of tyrosinyl-tRNA? A) Puromycin B) Streptomycin C) Actinomycin D D) Cyclohexamide Q122. The following drugs have activity against hepatitis B virus
  • 33. ORIGINAL COPY 33 of 40 1. Lamivudine 2. Emtricitabine 3. Tenofovir 4. Efavirenz A) Only 1, 2 and 3 are correct B) Only 1and 3 are correct C) Only 2 and 4 are correct D) Only 4 is correct Q123. A screening test for cancer has been evaluated in 400 people and results are shown below in table Screening test Cancer No cancer Total Positive 90 30 120 Negative 10 270 280 Total 100 300 400 The positive predictive value for the screening test is A) 90% B) 10% C) 75% D) 96% Q124. A 34-year-old man with HIV/AIDS (CD4+ T-lymphocyte count = 110/mm3 ) develops a scaly, waxy, yellowish, patchy, crusty, pruritic rash on and around his nose. The rest of his skin examination is normal. Which of the following is the most likely diagnosis? A) Molluscum contagiosum B) Seborrheic dermatitis C) Reactivation herpes zoster D) Psoriasis Q125. "Russell's" sign is seen in A) Anorexia Nervosa B) Bulimia Nervosa C) Somatization disorder D) Panic Disorder Q126. Electrolyte imbalance has to be regularly corrected in intensive care unit patients. Approximate daily requirement of potassium is
  • 34. ORIGINAL COPY 34 of 40 A) 10 milli moles B) 30 milli moles C) 50 milli moles D) 70 milli moles Q127. The Ultrasound of a G2 P1L1 done at 26 weeks, showed a "Double Bubble" sign in the fetus. The most probable diagnosis is A) Gastroschisis B) Omphalocoel C) Duodenal Atresia D) Polycystic Kidney Q128. True statement regarding nephrogenic fibrosing dermopathy associated with magnetic resonance contrast agent gadolinium A) Patient with stage 2 chronic kidney disease can be administered normal amount of gadolinium B) Associated facial fibrosis is a rule C) Concurrent liver disease is not a risk factor D) Gadolinium can be removed rapidly by hemodialysis Q129. In cases of closed lower limb injuries, it is always important to suspect compartment syndrome and to take preventive measures. Which of the following statement is wrong regarding compartment syndrome? A) Characterized by severe pain, distal sensory disturbance and absence of distal pulses. B) When in doubt of this situation, it is not safe to do fasciotomy C) Delayed fasciotomy may cause myoglobinuria and renal failure D) It can even occur in open injuries if the wound doesn't extend into the affected compartment. Q130. In the case of the ureters, the anatomical points of constriction are 1. At the ureteropelvic junction, just inferior to the kidney.
  • 35. ORIGINAL COPY 35 of 40 2. Where ureters cross the common iliac vessels at the pelvic brim. 3. Where ureters enter the wall of bladder. 4. At the renal sinuses. A) 1, 2, 3 are correct B) Only 1, 3 are correct C) Only 2, 4 are correct D) Only 4 is correct Q131. A small change in volume leads to an inordinate increase in intracranial pressure when A) Intracranial compliance is high B) Intracranial elastance is high C) Intracranial elastance is low D) Intracranial resistance is low Q132.A man with hemophilia A married a normal woman whose father had Hemophilia A. Regarding their children Assertion: 50% of female children will suffer from the disease Reason: The abnormality is inherited as X linked dominant A) Both Assertion and Reason are true, and Reason is the correct explanation for Assertion B) Both Assertion and Reason are true, and Reason is not the correct explanation for Assertion C) Assertion is true, but Reason is false D) Assertion is false, but Reason is true Q133. Regarding Modified Ritgens manoeuvre, Assertion: It allows controlled delivery of the head Reason: This position favours neck extension so that the head is delivered by the smallest diameter passing through the introitus A) Both Reason and Assertion are true and Reason is the correct explanation for Assertion B) Both Assertion and Reason are true and Reason is not the correct explanation for the Assertion C) Assertion is true but Reason is false D) Assertion is false but Reason is true. Q134. In the process of secondary active transport involved in molecule movement across cell membranes A) Energy expenditure is minimal
  • 36. ORIGINAL COPY 36 of 40 B) Transport of Na + is coupled to that of other molecules C) Osmotic gradient is established across cells D) Size of intercellular junction is a limiting factor Q135. Jerger's curve on impedance audiometry in post stapedotomy case will be A) B Type B) Ad type C) As type D) C type Q136. A 45 year old lady, a hypertensive came with complaints of partial visual deficits and her fundus picture is given below The most likely diagnosis is A) Papillitis B) Branched retinal vein occlusion C) Retinitis pigmentosa D) Central retinal artery occlusion Q137. Which of the following drug acts by inhibiting the enzyme Inosine monophosphate dehydrogenase A) Azathioprine B) Alefacept C) Mycophenolate mofetil D) Pimecrolimus Q138. PCOD (Poly Cystic Ovarian Disease) in earlier life may be attributed to which of the following in later life 1. Hypertension
  • 37. ORIGINAL COPY 37 of 40 2. Type 2 diabetes mellitus 3. Endometrial carcinoma 4. Ovarian carcinoma A) Only 1, 2 and 3 are correct. B) Only 1, 3 are correct C) Only 2, 4 are correct D) Only 4 is correct Q139. Jefferson fracture is- A) Fracture of atlas ring caused by flexion B) Fracture of atlas ring caused by axial loading C) Fracture of axis ring caused by flexion D) Fracture of axis ring caused by axial loading Q140. A 12-year old child had a sore-throat, following which, he presented with fever, malaise, nausea, oliguria and smoky urine. BP was 130/90 mm of Hg. Urine examination revealed proteinuria (750 mg/day) and red cell casts. The renal biopsy revealed hypercellular glomeruli infiltrated by neutrophils with proliferation of endothelial cells and mesangial cells. The likely diagnosis would be A) Minimal-change disease B) Post-infectious glomerulonephritis C) Alport syndrome D) Good Pasture syndrome Q141. At autopsy, massive splenomegaly was detected, with gross appearance of "sago spleen". Microscopic sections reveal deposition of amorphous, eosinophilic material, limited to the splenic follicles .The likely diagnosis would be A) Miliary tuberculosis B) Amyloidosis C) Septic abscesses D) Non-Hodgkins lymphoma Q142. Bouchard's Nodes are seen in A) Meta - Carpo Phalangeal joints
  • 38. ORIGINAL COPY 38 of 40 B) Distal inter- Phalangeal joints C) Proximal Inter- Phalangeal joints D) Carpo - Meta carpal joints. Q143. Regarding ESBL producing strains of Gram negative bacilli, Assertion: Are resistant to multiple antibiotic groups and may include fluoroquinolones, aminoglycosides, beta lactamase inhibitor combinations and carbapenems. Reason: A single determinant on a resistance plasmid codes for resistance to all the groups of antibiotics. A) Both Assertion and Reason are true, and Reason is the correct explanation for Assertion B) Both Assertion and Reason are true, and Reason is not the correct explanation for Assertion C) Assertion is true, but Reason is false D) Assertion is false, but Reason is true Q144. Which of the following hormone uses phospholipase C as second messenger A) Secretin B) Oxytocin C) Calcitonin D) Somatostatin Q145. Calot's triangle or hepatobiliary triangle is an important landmark for hepatobiliary surgeries. Its boundaries are formed by: A) Cystic duct, right hepatic duct and common hepatic duct B) Common bile duct, cystic duct and cystic artery C) Cystic duct, common bile duct and duodenum D) Cystic duct, cystic artery and common hepatic artery Q146. The virus known to be associated with nasopharyngeal carcinoma is A) Cytomegalovirus B) Hepatitis B virus C) Hepatitis C virus D) Epstein Barr virus Q147. Which of the following terms is not used to describe pathology in rheumatic heart disease?
  • 39. ORIGINAL COPY 39 of 40 A) Rokitansky-Aschoff sinuses B) Aschoff bodies C) Anitschkow cells D) Mac Callum plaques Q148. Shown below is the distribution of the laboratory test results in normal and two different disease states (disease 1 and disease 2). Assertion: The laboratory test is more reliable to diagnose disease 1 than the disease 2 Reason: Test reliability decreases if test result values in diseased population overlap with those of the normal population A) Assertion and Reason are correct and Reason is the correct explanation of the Assertion B) Assertion and Reason are correct and Reason is not the correct explanation of the Assertion C) Assertion is true and Reason is false D) Assertion is false and Reason is true Q149. Key features of biologic agents used as bioweapons are 1. High morbidity and mortality rates. 2. High infective dose 3. Highly infectious by aerosol. 4. No potential for person to person spread A) Only 1, 2 and 3 are correct. B) Only 1 and 3 are correct C) Only 2 and 4 are correct D) Only 4 is correct Q150. Cell of origin of meningioma is A) Oligodendrocyte
  • 40. ORIGINAL COPY 40 of 40 B) Arachnoid cap cell C) Neuroblast D) Pial cell